You are on page 1of 102

www.insightsonindia.

com

TEST 1 Solutions

www.insightsias.com

Prelims 2017 Test 1


SOLUTIONS

1. As the rulers of the mahajanapadas of the Common Era were building
huge forts maintaining big armies, they needed more resources for
which they collected taxes. Consider the following about it.
1. Taxes on crop were fixed at a certain share of the total produce.
2. Craft persons were made to pay taxes.
3. Hunter gatherers and herders were exempt from taxes as it was
not possible to enumerate them as a part of any specific empire.
4. Trade was tax-free but a surcharge was levied on special category
of goods.
Select the correct answer using the codes below.
a)
b)
c)
d)

1 and 2 only
1, 3 and 4 only
2 and 3 only
1, 2, 3 and 4

Solution: a)
Justification: Statement 1: Taxes on crops were the most important. This
was because most people were farmers. Usually, the tax was fixed at 1/6th
of what was produced. This was known as bhaga or a share.
Statement 2: These could have been in the form of labour. For example, a
weaver or a smith may have had to work for a day every month for the
king.
Statement 3: Herders were also expected to pay taxes in the form of
animals and animal produce. And hunters and gatherers also had to
provide forest produce to the raja. So, 3 is incorrect.
Statement 4: There were also taxes on goods that were bought and sold,
through trade. So, 4 is also incorrect.
Q Source: Page 59: NCERT 6th History: Our Pasts-I

INSIGHTS PRELIMS TEST SERIES 2017

www.insightsonindia.com

TEST 1 Solutions

www.insightsias.com



2. Sariska Tiger Reserve is known for
a) Being the first Tiger reserve in the World to have successfully
relocated tigers
b) Confluence of four rivers at a single location harbouring one of the
richest biodiversity of predators in India
c) Successfully stopping poaching of tigers and other related species
completely
d) Being the first Tiger Reserve to be named in CITES agreement

Solution: a)
Justification: In 2004, it was found that no tigers were left in Sariska
reserve, which the Rajasthan Forest Department largely attributed initially
to migration, but later to poaching.
The matter was taken seriously, and in 2005, the Government of Rajasthan,
in cooperation with the Government of India and Wildlife Institute of India
(WII), planned the re-introduction of tigers to Sariska from Ranthambore
tiger reserve.
It thus became the first tiger reserve in the world to have successfully
relocated tigers.
Learning: This area was a hunting preserve of the Alwar state and it was
declared a wildlife reserve in 1955.
The area of Sariska, being a part of the Aravalli Range, is rich in
mineral resources, such as copper.
The reserve is also the location of several sites of historical
importance such as the 16th-century Kankwadi fort, originally built
by Jai Singh II, located near the centre of the park.
Pandupol in the hills in the centre of the reserve is believed to be one
of the retreats of Pandava.

INSIGHTS PRELIMS TEST SERIES 2017

www.insightsonindia.com

TEST 1 Solutions

www.insightsias.com

Q Source: Page 63 (news reports): NCERT 6th Geography: The Earth Our
Habitat



3. The mid-day Sun will NEVER be exactly overhead on which of the
following locations?
1. Any latitude in Polar Regions
2. Any latitude where equatorial vegetation is found
3. Any longitude that touches poles
4. Any longitude that passes through Scandinavian countries
Select the correct answer using the codes below.
a)
b)
c)
d)

1 and 3 only
2, 3 and 4 only
1, 3 and 4 only
1 only

Solution: d)
Concept: The mid-day sun is exactly overhead at least once a year on all
latitudes in between the Tropic of Cancer and the Tropic of Capricorn. This
area, therefore, receives the maximum heat and is called the Torrid Zone.
The mid-day sun never shines overhead on any latitude beyond the Tropic
of Cancer and the Tropic of Capricorn. The angle of the suns rays goes on
decreasing towards the poles.
Justification: Statement 1: As evident from the explanation above, 1 is
clearly an answer.
Statement 2: Equatorial vegetation can be found near equator, where Suns
rays are generally overhead.
Statement 3: Every longitude starts from a pole and meets the other pole. It
is latitude that matters in distribution of solar insolation, not the longitude.

INSIGHTS PRELIMS TEST SERIES 2017

www.insightsonindia.com

TEST 1 Solutions

www.insightsias.com

Statement 4: Based on the same logic as above, 4 is also incorrect.


Q Source: Page 12: NCERT 6th Geography: The Earth Our Habitat



4. What is the difference between the animals Changthangi and Chiru?
a) Changthangi lives in hot and arid areas, whereas Chiru is adapted to
live in semi-desert areas of cold high mountains
b) Changthangi is an antelope, whereas Chiru is a primate.
c) Changthangi is famously reared for Pashmina wool, whereas Chiru is
reared for getting Shahtoosh wool
d) Changthangi is poached for its horns, whereas Chiru is hunted for its
musk which has great medicinal value

Solution: c)
Justification: Option A: The Changthangi or Pashmina goat inhabits the
plateaus in Tibet and neighbouring areas of Ladakh in Jammu & Kashmir.
They are raised for ultra-fine cashmere wool (pashmina).
Tibetan Antelope of Chiru is native to Tibet.
Option B: Chiru is an antelope, whereas Changthangi is a goat. Primate are
apes, humans etc.
Option C-D: In recent years Chiru has become endangered due to poaching
for their soft and warm wool which is usually obtained after death. This
wool is known as shahtoosh and is used to weave shawls . At present,
international trade in their products is strictly prohibited.
Q Source: Improvisation: Page 75: Geography NCERT 7th: Our Environment


INSIGHTS PRELIMS TEST SERIES 2017

www.insightsonindia.com

TEST 1 Solutions

www.insightsias.com

5. Why 15th August was chosen as the Independence Date of India?


a) It was on this date that Indian Independence Bill was introduced in
the British House of Commons and passed.
b) Lord Mountbatten chose the date as he considered this date to be
lucky.
c) It was on this date that the Lahore Session of the Congress passed the
Purna Swaraj resolution.
d) On this date, Indias Constituent Assembly passed the Objectives
Resolution granting freedom to India.

Solution: b)
Justification & Learning: Option A: Indian Independence Bill was
introduced in the British House of Commons on July 4, 1947 and passed
within a fortnight.
Option B: The date was chosen by Lord Mountbatten himself because he
had considered this date to be lucky. It was on this day during the World
War II, that the Japanese Army surrendered to the allies.
Option C: In 1929, when Jawaharlal Nehru as Congress President gave the
call for Poorna Swaraj or total independence from British colonial rule,
January 26 was chosen as the Independence Day. Later it was declared as
the Republic Day.
Q Source: http://indianexpress.com/article/india/india-news-india/whywas-august-15-chosen-as-independence-day/

6. NASA's Fermi Mission is aimed at understanding
1. Earths geological structure
2. Role of exo-planets in providing a habitable space on earth
3. Search for signs of new laws of physics and what composes the
mysterious Dark Matter
Select the correct answer using the codes below.
a) 1 only
b) 2 and 3 only
c) 3 only

INSIGHTS PRELIMS TEST SERIES 2017

www.insightsonindia.com

TEST 1 Solutions

www.insightsias.com

d) None of the above

Solution: c)
Justification: NASA's Fermi (formerly Gamma-ray Large Area Space
Telescope, or GLAST) is a powerful space observatory (from 2008) that has
started to:
Explore the most extreme environments in the Universe, where
nature harnesses energies far beyond anything possible on Earth.
Search for signs of new laws of physics and what composes the
mysterious Dark Matter.
Explain how black holes accelerate immense jets of material to nearly
light speed.
Help crack the mysteries of the stupendously powerful explosions
known as gamma-ray bursts.
Answer long-standing questions across a broad range of topics,
including solar flares, pulsars and the origin of cosmic rays.
Learning: Gamma rays are the highest-energy form of light, and the
gamma-ray sky is spectacularly different from the one we perceive with
our own eyes.
Fermi data has enabled scientists to answer persistent questions across a
broad range of topics, including supermassive black-hole systems, pulsars,
the origin of cosmic rays, and searches for signals of new physics.
Q Source: Current Affairs: NASA Website:
https://www.nasa.gov/feature/goddard/2016/nasas-fermi-missionexpands-its-search-for-dark-matter

7. Consider the following about spot markets in India.
1. They are electronic financial markets where on spot deals for
futures in commodities are fixed.
2. National Spot Exchange Limited (NSEL) is the only recognized
spot market in India.
3. They are regulated by the Securities and Exchange Board of India
(SEBI).

INSIGHTS PRELIMS TEST SERIES 2017

www.insightsonindia.com

TEST 1 Solutions

www.insightsias.com

Select the correct answer using the codes below.


a)
b)
c)
d)

1 and 2 only
1 and 3 only
3 only
1, 2 and 3

Solution: c)
Justification: Statement 1: It is a public financial market in which financial
instruments or commodities are traded for immediate delivery. It
contrasts with a futures market, in which delivery is due at a later date. So,
1 is incorrect.
Statement 2: There are many players in the sport market, for e.g. NCDEX
Spot Exchange Limited, Indian Bullion Spot Exchange limited. NSEL is only
one of them, which was also in news due to a scam.
Statement 3: Spot Exchange is presently recognized by Ministry of
Consumer Affairs, Food & Public Distribution. It was regulated by the
Forward Contracts Act earlier by Forward Markets Commission. Now, since
FMC has been merged with SEBI, SEBI regulates it.
Learning: Current events section explains the NSEL scam very well
http://www.insightsonindia.com/2016/08/01/insights-daily-currentaffairs-01-august-2016/
You can also go through this to know more about spot markets
http://www.arthapedia.in/index.php?title=Spot_Exchanges
Q Source: NSEL Scam



8. The Constitution (122nd Amendment) (GST) Bill, 2014 must be
passed by at least half of all state assemblies in India by
a) Two-third majority of the assembly present and voting
b) Absolute majority of the total membership of the house

INSIGHTS PRELIMS TEST SERIES 2017

www.insightsonindia.com

TEST 1 Solutions

www.insightsias.com

c) Simple majority in form of a resolution


d) Concurrence of the Speaker of the House and agreed to by the
Governor of the State

Solution: c)
Learning: Amendments that seek to change federal provisions of the
Constitution will be first passed in the two houses separately by absolute
and special majority.
They afterwards also need to be ratified by legislatures of at least half of
the states by resolutions.
Only after this, the bill will be sent for presidential assent, and it becomes
an Act.
Assam recently became the first state to ratify the GST Bill.
Q Source: http://www.thehindu.com/news/national/other-states/assambecomes-first-state-to-ratify-gst-bill/article8980777.ece



9. With regard to pollution and climate change, why environment
regulators are targeting Diesel cars more than Petrol cars?
1. Diesel cars have higher emissions of NOx and particulate matter
than petrol cars.
2. High sulphur content emitted by Diesel cars lead to acid rain.
Select the correct answer using the codes below.
a)
b)
c)
d)

1 only
2 only
Both 1 and 2
None

Solution: c)

INSIGHTS PRELIMS TEST SERIES 2017

www.insightsonindia.com

TEST 1 Solutions

www.insightsias.com

Concept: Naturally occurring crude can be burnt directly to produce


energy, but because different hydrocarbons have different boiling points,
the combustion will be very uneven.
As such, crude oil is refined into compounds like kerosene, LPG,
paraffin wax, naphtha, etc., apart from petrol and diesel.
Longer the carbon chain, higher is the probability of finding other
elements like sulphur and nitrogen attached to it. Petrol is a more
uniform carbon compound than diesel as petrol has shorter chains
than Diesel.
Crucially, diesel has higher sulphur content.
Justification: Statement 2: Emissions from diesel vehicles, especially
particulate matter and black carbon, are the major reasons for air pollution
as well. Sulphates, like sulphur dioxide, released in the atmosphere may
lead to acid rain. So, statement 2 is correct.
Petrol exhaust produces carbon monoxide (CO) and some nitrous oxides
(NOx) during combustion.
Statement 1: The CO combines with oxygen again to produce carbon
dioxide (CO2), which is what comes out of the exhaust pipes of petrol
vehicles.
The relatively high nitrogen and sulphur content in diesel results in
the formation of higher amounts of NOx, and particulate matter
containing mainly sulphurous compounds.
CO2 is a greenhouse gas, and contributes to global warming; NOx and
particulate matter carry major health risks.
Q Source: Improvisation: http://www.downtoearth.org.in/news/scallows-diesel-suvs-back-into-the-capital-55281



10.
Which of the following is NOT vital for the establishment or
success of a democracy?

INSIGHTS PRELIMS TEST SERIES 2017

www.insightsonindia.com

a)
b)
c)
d)

TEST 1 Solutions

www.insightsias.com

Presence of Rule of Law


Granting fundamental rights to the population
Political equality of citizens to run for public offices
Direct elections of all representatives

Solution: d)
Justification: Option A: Rule of law ensures that democracy doesnt turn
into Rule of Men.
Option B: Fundamental rights guard the citizens from the tyranny and
despotism of the rulers.
Option C: If a certain class of citizens are banned (without any intelligible
criteria) from participating in elections or running for public offices, a
democracy may not be sustained.
Option D: Many democracies practice indirect election of representatives,
for e.g. President of USA is indirectly elected. Voters elect the Electoral
College, which then elects the President.
In India, Rajya Sabha MPs are indirectly elected. So, direct election of all
representatives is not crucial for a democracy.
Q Source: Page 32: NCERT 6th: Social and Economic Life I



11.
In India, the public health service is a chain of health centres
and hospitals run by the government. Under it, Primary Health
Centres (PHCs) serve
a) Many villages in a rural area
b) Specific wards in a Municipality
c) Several hundreds of villages at the block level
d) As attachments of the District Hospitals in sub-urban areas

Solution: a)

INSIGHTS PRELIMS TEST SERIES 2017

www.insightsonindia.com

TEST 1 Solutions

www.insightsias.com

Learning: At the village level there are sub-health centres where there is
usually a nurse and a village health worker. They are trained in dealing
with common illnesses.
They work under the supervision of doctors at the Primary Health Centre
(PHC) which is next in the hierarchy. Such a centre covers many villages in
a rural area.
At the Block level, there is usually a Community Health Centre with more
facilities than at PHC.
At the district level is the District Hospital that also supervises all the
health centres.
Q Source: Page 24: NCERT 7th: Social and Economic Life II



12.
Which of the following clearly shows the importance of carbon
dioxide in earths atmosphere?
1. It is essential for the growth of plants.
2. It is the most abundant gas in the atmosphere after nitrogen and
oxygen.
3. It absorbs the heat directly incoming from the Sun thus warming
the planet.
4. Ocean absorbs carbon dioxide from the atmosphere to form
carbonate rocks.
Select the correct answer using the codes below.
a)
b)
c)
d)

1 and 4 only
2 and 3 only
1, 3 and 4 only
1, 2, 3 and 4

Solution: a)

INSIGHTS PRELIMS TEST SERIES 2017

www.insightsonindia.com

TEST 1 Solutions

www.insightsias.com

Justification: Statement 1 : Green plants use CO2 gas to prepare their food
through photosynthesis.
Statement 2: Argon is the next abundant gas.


Statement 3: Carbon dioxide in the atmosphere does not absorb the
incoming visible radiations coming from the sun blocks the out going
infrared radiations radiated by earth. By absorbing infrared radiations, the
atmosphere gets heated. This is known as Green house Effect. The heated
atmosphere keeps the earthworm. Thus carbon dioxide helps in keeping
the earth warmer by blocking the infrared radiations.
Statement 4: When carbon dioxide (CO2) is absorbed by seawater,
chemical reactions occur that reduce seawater pH, carbonate ion
concentration, and saturation states of biologically important calcium
carbonate minerals.
It is called "ocean acidification".
Calcium carbonate minerals are the building blocks for the skeletons
and shells of many marine organisms.
Learning: Volcanic outgassing and wildfires are two significant natural
sources of CO2 in Earth's atmosphere. Respiration, the process by which
organisms liberate energy from food, emits carbon dioxide.
Carbon dioxide is the most abundant gas in the atmospheres of Mars and
Venus.

INSIGHTS PRELIMS TEST SERIES 2017

www.insightsonindia.com

TEST 1 Solutions

www.insightsias.com

Q Source: Improvisation: Page 35: NCERT 6th Geography: The Earth Our
Habitat




13.
Consider the following about Unstructured Supplementary
Service Data (USSD) that is being utilized by many corporations in
India for service delivery.
1. Use of USSD is limited due to its requirement of high speed
internet.
2. It can be used for serving the banking needs of customers.
Which of the above is/are correct?
a)
b)
c)
d)

1 only
2 only
Both 1 and 2
None

Solution: b)
Justification: USSD is a GSM based technology i.e. used to send text
between a mobile phone and an application program in the network.
Unlike Short Message Service (SMS) messages, USSD messages create a
real-time connection during a USSD session. So they are more responsive
than services that use SMS.
Statement 1: So, it can be used even without any internet connection as it
does not require any data. 1 will be wrong.
Statement 2: The Union Bank of India (UBI) has recently launched an
Unstructured Supplementary Service Data (USSD) based *99# mobile
application for their customers for basic banking needs.

INSIGHTS PRELIMS TEST SERIES 2017

www.insightsonindia.com

TEST 1 Solutions

www.insightsias.com

It was launched in partnership with National Payments Corporation of


India (NPCI), the umbrella organisation for all retail payments system in
the country.
Q Source:
http://economictimes.indiatimes.com/industry/banking/finance/banking
/-union-bank-of-india-launches-ussd-based-mobile-app-for-basic-bankingneeds-with-npci/articleshow/53673011.cms


14.
The Archaeological Ruins of the location comprise the first
great urban centre of the Indus civilization with burnt brick
structures. It is a UNESCO World Heritage Site and recently a movie
on it was directed by Ashutosh Gowariker. It is?
a) Burzahom
b) Inamgaon
c) Lothal
d) None of the above

Solution: d)
Learning: It is Mohenjodaro. The Archaeological Ruins at Moenjodaro are
the best preserved urban settlement in South Asia dating back to the
beginning of the 3rd millennium BC, and exercised a considerable influence
on the subsequent development of urbanization.
The archaeological ruins are located on the right bank of the Indus
River
It was one of the largest settlements of the ancient Indus Valley
Civilization.
The Archaeological Ruins at Moenjodaro are being protected by
National and Regional laws including the Antiquities Act 1975
The discovery of Moenjodaro in 1922 revealed evidence of the
customs, art, religion and administrative abilities of its inhabitants.

INSIGHTS PRELIMS TEST SERIES 2017

www.insightsonindia.com

TEST 1 Solutions

www.insightsias.com


Q Source: Recent movie + Improvisation: Chapter 4: NCERT 6th History:
Our Pasts-I



15.
Which of the following forest types covers the largest area in
India?
a) Tropical Moist Deciduous Forest
b) Tropical Evergreen Forests
c) Littoral and Swamp Forest
d) Sub-Tropical Broad Leaved Hill Forest

Solution: a)

INSIGHTS PRELIMS TEST SERIES 2017

www.insightsonindia.com

TEST 1 Solutions

www.insightsias.com

Justification: FAO has provided the relevant statistics, where largest


Tropical moist deciduous is followed by tropical dry deciduous. So,
deciduous forests cover the largest forest area in India, more than 60%!
Tropical wet evergreen forests follow and then semi-evergreen forests.


Q Source: Improvisation: Page 59: NCERT 6th Geography: The Earth Our
Habitat



16.
In India, rocks of which of the following regions was formed the
earliest?
a) Khasi hills
b) Lower Shiwaliks
c) Indo-Gangetic plain
d) Deccan Traps

Solution: d)
Justification: Geoglogy of India can be categorized in following eras, in
increasing chronological order.

INSIGHTS PRELIMS TEST SERIES 2017

www.insightsonindia.com

TEST 1 Solutions

www.insightsias.com

Precambrian super-eon
Phanerozoic
o Palaeozoic
Lower Paleozoic
Upper Paleozoic
o Mesozoic
o Cenozoic
Tertiary period
Quaternary period
Option (a): Khasi hills, one of youngest, belong to Tertiary period (to which
a large area in North-east also belongs).
Option (b): Shiwaliks are part of Himalayas, so given the reasoning below,
this (b) will also be incorrect.
Option (c): The alluvium which is found in the Indo-Gangetic plain belongs
to the Quaternary period. It was eroded from the Himalayas by the rivers
and the monsoons. Since Himalayas were formed after Deccan traps, and
are very young, (c) cant be the answer.
The Deccan plateau in India is one of the oldest plateaus in India made
mainly from igneous rocks.
We will be covering more questions on this area in later tests.
Q Source: Improvisation: Page 42: NCERT 6th Geography: The Earth Our
Habitat



17.
Consider the following about the United Nations Commission
on Sustainable Development (CSD).

INSIGHTS PRELIMS TEST SERIES 2017

www.insightsonindia.com

TEST 1 Solutions

www.insightsias.com

1. It was established by the Conference of Parties (COP) to the


UNFCCC.
2. Its primary duty is to enforce the binding agreements signed
under the UNFCCC.
3. It meets every ten years to chart the course of sustainable
development in both developed and developing countries.
Select the correct answer using the codes below.
a)
b)
c)
d)

1 only
2 and 3 only
3 only
None of the above

Solution: d)
Justification: Statement 1: It was established by the UN General Assembly
in December 1992 to ensure effective follow-up of United Nations
Conference on Environment and Development (UNCED), also known as the
Earth Summit.
At the United Nations Conference on Sustainable Development (Rio+20),
Member States agreed to establish a high level political forum that replaced
the Commission on Sustainable Development. The body no longer exists.
Statement 2: The Commission was responsible for reviewing progress in
the implementation of Agenda 21 and the Rio Declaration on Environment
and Development; as well as providing policy guidance to follow up the
Johannesburg Plan of Implementation (JPOI) at the local, national, regional
and international levels.
Statement 3: The CSD met annually in New York, in two-year cycles,
opening its sessions to broad participation from both governmental and
non-governmental actors.
Q Source: https://sustainabledevelopment.un.org/intergovernmental/csd

INSIGHTS PRELIMS TEST SERIES 2017

www.insightsonindia.com

TEST 1 Solutions

www.insightsias.com


18.
Non-performing Assets (NPAs) are loans made by a bank or
finance company on which repayments or interest payments are not
being made on time. How do high NPAs affect the citizens
(customers) in India?
1. They may lead to an increase in the cost of capital for borrowers.
2. They may lead to bank defaults and thus lower peoples
confidence in the banking system affecting their savings ratio.
Which of the above is/are correct?
a)
b)
c)
d)

1 only
2 only
Both 1 and 2
None

Solution: c)
Justification: Statement 1: It results in inflating the cost of capital for
economic activities and banks may charge higher interest rates on some
products to compensate NPAs.
NPA is any asset of a bank which is not producing any income. It affects the
profitability & liquidity of the banks.
Statement 2: It adversely affects the value of bank in terms of market
credit and widens assets and liability mismatch. This may even lead to bank
defaults.
It shakes consumers confidence in the banking deposits, and forces them
to withdraw savings in banks, thus lowering the savings ratio.
Learning: Savings ratio is the total amount of a persons income not spent
on consumption and invested in either bank deposits, bonds or other
market instruments.
In any economy, savings finance investments. Higher savings increase
liquidity in the market and reduce the cost of credit resulting in higher
economic growth.

INSIGHTS PRELIMS TEST SERIES 2017

www.insightsonindia.com

TEST 1 Solutions

www.insightsias.com

So, it is very important that the banking system of an economy is robust.


Q Source: http://www.thehindubusinessline.com/opinion/npas-banksalone-are-not-to-blame/article8955877.ece



19.
If we did NOT use an Indian Standard Time, the actual time
difference between which of the following places would be more than
1 hour?
a) Patna and Kolkata
b) Dwarka and Dibrugarh
c) Chandigarh and Rewari
d) Sultanpur and Kanpur

Solution: b)
Justification: Sultanpur and Kanpur both are in UP, Chandigarh and
Rewari both in Haryana; Patna and Kolkata situated not far by.
The difference in time is created by longitude differences. Difference of 15
degrees creates a difference of 1 hour.
Based on the longitudes of Dwarka (Gujarat) and Dibrugarh (Assam), there
will be a difference of about 1 hour and 45 minutes in the local times of
Dwarka and Dibrugarh.
Q Source: Page 15: NCERT 6th Geography: The Earth Our Habitat



20.
Which of the following correctly differentiates the events of
equinox with that of solstice?

INSIGHTS PRELIMS TEST SERIES 2017

www.insightsonindia.com

TEST 1 Solutions

www.insightsias.com

1. In a solstice only a particular tropic receives direct vertical rays of


the Sun unlike in an equinox the equator receives direct vertical
rays of Sun.
2. Solstice is caused only due to rotation of the earth, whereas
equinox is caused only due to revolution of the earth.
Which of the above is/are correct?
a)
b)
c)
d)

1 only
2 only
Both 1 and 2
None

Solution: a)
Justification: Statement 1: The image below explains statement 1. Notice
how on Summer equinox, Tropic of Cancer receives direct overhead rays of
the Sun, while in Winter solstice, it is the Tropic of Capricorn.
On an equinox, when days and nights are equal in duration, Sun is directly
overhead the equator, causing equal days and nights.


Statement 2: Both are caused due to axial tilt of the earth, its revolution
and also rotation (to cause day and night). The diagram above makes it
clear.
Q Source: Page 19: NCERT 6th Geography: The Earth Our Habitat


INSIGHTS PRELIMS TEST SERIES 2017

www.insightsonindia.com

TEST 1 Solutions

www.insightsias.com



21.
Consider the following about the Panchayat Secretary.
1. She is elected by the Gram Panchayat from amongst its members.
2. She is responsible for calling the meeting of the Gram Sabha and
Gram Panchayat and keeping a record of the proceedings.
Which of the above is/are correct?
a)
b)
c)
d)

1 only
2 only
Both 1 and 2
None

Solution: b)
Justification: Statement 1: She is not an elected person but is appointed by
the government.
Statement 2: Role of Panchayat Secretary is defined by State
rules/regulations and laws. In the state of AP, some of her roles are:
She shall maintain the required registers of the Gram Panchayat.
She is subordinate to the Gram Panchayat and shall function under
the control of the Sarpanch of the Gram Panchayat.
She shall attend the meeting of the Gram Panchayat or any committee
thereon.
She shall take the responsibility of the protection of Panchayat lands
and properties.
She shall report of encroachments, damage or misuse of Panchayat
lands and buildings if any to the higher authorities.
She shall assist Grama Sabha in the identification of beneficiaries,
disbursement of loans and their recoveries.
Q Source: Page 44: NCERT 6th: Social and Economic Life I

INSIGHTS PRELIMS TEST SERIES 2017

www.insightsonindia.com

TEST 1 Solutions

www.insightsias.com




22.
The Arctic Ocean is located within the Arctic Circle and
surrounds the North Pole. It is connected with the Pacific Ocean by a
narrow stretch of shallow water known as
a) Berring strait
b) Alaska Strait
c) Isthmus of Faribanks
d) Norton Sound

Solution: a)
Justification: It connects the Pacific and Arctic oceans.
The International Date Line runs around the Bering strait.
This strait separates the United States and Russia by around 85 km, with a
water depth that measures only 3050 meters!
In the last few decades some factions have discussed the construction of a
bridge over the strait, however, financial and weather concerns have
continually stalled the project.

INSIGHTS PRELIMS TEST SERIES 2017

www.insightsonindia.com

TEST 1 Solutions

www.insightsias.com


Q Source: Page 35: NCERT 6th Geography: The Earth Our Habitat




23.
Communicable diseases account for a large burden of diseases
in India. Which of these may NOT help controlling their spread?
a) Improving sanitation and drinking water in the locality
b) Isolating infected persons
c) Reducing intake of salt in food
d) Using only new syringes for blood transfusion and disposing old used
ones

Solution: c)
Learning: Communicable diseases are caused by pathogens passed from
one human to another.

INSIGHTS PRELIMS TEST SERIES 2017

www.insightsonindia.com

TEST 1 Solutions

www.insightsias.com

Pathogens are viral, bacterial, parasitic and fungal. Methods of transmission


include mucus, blood, breath, saliva and sexual contact.
Contaminated surfaces, such as doorknobs, counter tops and playground
equipment, provide a medium for passing disease from one human to
another.
So, all options other than C are correct.
Option C can control cardiovascular non-communicable diseases.
Q Source: Page 29: NCERT 7th: Social and Economic Life II



24.
Which of the following factors contribute to the evolution of
landforms on earth?
1. Movement of magma within the earth
2. Growth and decay of vegetation
3. Erosion and deposition
4. Frost action
Select the correct answer using the codes below.
a)
b)
c)
d)

1 and 3 only
2 and 4 only
1, 3 and 4 only
1, 2, 3 and 4

Solution: d)
Justification: Statement 1: It causes plate movements (tectonics), results in
volcanic eruptions and is therefore a significant factor in the evolution of
landforms on earth.
Statement 2: If Vegetation cover is high, it protects the surface from rain
splash as root mass is sufficient to stabilize the materials on the slope.

INSIGHTS PRELIMS TEST SERIES 2017

www.insightsonindia.com

TEST 1 Solutions

www.insightsias.com

Even surface runoff become less effective in carving out landforms in areas
where there is dense vegetation, since there is little scope for soil erosion.
Statement 3: Self-explanatory. For e.g. all the plains that you are the result
of active erosion and deposition by water.
Statement 4: When water freezes to ice, its volume increases. Under
specific circumstances, this expansion is able to displace or fracture rocks
where water exist in its pores. Repeated frost action thus weathers
(breaks) the rocks.
In some mountains, there are permanently frozen rivers of ice. They
are called glaciers. Glaciers move at a very slow rate. When they do,
they erode the soil beneath them.
Also, formation of glaciers and their retreat affects the soil profile of
the region and thus the landforms.
Q Source: Page 39-40: NCERT 6th Geography: The Earth Our Habitat




25.
In India, women got universal suffrage rights with the
st
a) 61 Constitutional Amendment, 1989
b) Enactment of Government of India Act 1935
c) Commencement of the Indian Constitution
d) Act of 1858 passed by the British Parliament

Solution: c)
Justification: Option A: The 61st Amendment Act lowered the voting age
from 21 to 18 years thus introducing adult suffrage. But, women already
enjoyed the right to vote in India.
Option C: The constitution provided for universal franchise (21 years age
or later) without discrimination of caste, creed or gender.

INSIGHTS PRELIMS TEST SERIES 2017

www.insightsonindia.com

TEST 1 Solutions

www.insightsias.com

Some British Acts did increase the level of electoral participation in India,
but they never introduced universal adult franchise. We will be covering
such Acts in the coming tests.
Q Source: Page 32: NCERT 6th: Social and Economic Life I


26.
Northern blot is a laboratory method used to
a) Determine the hereditary lineages in forensic investigation
b) Examine changes in plant physiology
c) Copy the information from one strand of genome to the other
d) Detect specific RNA molecules among a mixture of RNA

Solution: d)
Justification: Northern blotting can be used to analyze a sample of RNA
from a particular tissue or cell type in order to measure the RNA
expression of particular genes.
Application: Northern blotting allows one to observe a particular gene's
expression pattern between tissues, organs, developmental stages,
environmental stress levels, pathogen infection, and over the course of
treatment.
Learning: Some other related terms are:
A microarray is a laboratory tool used to detect the expression of
thousands of genes at the same time.
A proteome is the complete set of proteins expressed by an organism.
Q Source: Nature.com + http://www.news-medical.net/lifesciences/Northern-Blot-RNA-Blot.aspx + Related to UPSC 2016 Prelims
question on Transcriptome
https://sustainabledevelopment.un.org/frameworks/addisababaactionagenda

INSIGHTS PRELIMS TEST SERIES 2017

www.insightsonindia.com

TEST 1 Solutions

www.insightsias.com


27.
The State of Madhya Pradesh lying in Central India shares land
boundary with which of the following states?
1. Bihar
2. Jharkhand
3. Uttar Pradesh
4. Telangana
5. Odisha
Select the correct answer using the codes below.
a)
b)
c)
d)

1, 2 and 3 only
3 only
3, 4 and 5 only
2 and 4 only

Solution: b)
Justification: Both Bihar and Jharkhand dont touch MP, as UP land
boundary comes before that.
Extent of Maharashtra should be noted carefully. Many map based
questions use the extent of MH and UP to confuse you in state land borders
and their extent.


Q Source: Map based questions: India


INSIGHTS PRELIMS TEST SERIES 2017

www.insightsonindia.com

TEST 1 Solutions

www.insightsias.com



28.
Which of the following statements about the Financial Action
Task Force (FATF), established in 1989 by G-7, is INCORRECT?
a) It is an inter-governmental policy making body.
b) Its mandate covers setting international standards for combating
money laundering and terrorist financing.
c) It does not deal with proliferation of weapons of mass destruction
which are dealt by the UNSC and multi-lateral export control regimes.
d) It works to identify national-level vulnerabilities with the aim of
protecting the international financial system from misuse.

Solution: c)
Justification: Option B: The objectives of the FATF are to set standards and
promote effective implementation of legal, regulatory and operational
measures for combating money laundering, terrorist financing and other
related threats to the integrity of the international financial system.
The FATF is therefore a policy-making body which works to
generate the necessary political will to bring about national
legislative and regulatory reforms in these areas.
Option C: The FATF has developed a series of Recommendations that are
recognised as the international standard for combating of money
laundering and the financing of terrorism and proliferation of weapons of
mass destruction
Option D: The FATF monitors the progress of its members in implementing
necessary measures, reviews money laundering and terrorist financing
techniques and counter-measures, and promotes the adoption and
implementation of appropriate measures globally.
In collaboration with other international stakeholders, the FATF
works to identify national-level vulnerabilities with the aim of
protecting the international financial system from misuse.

INSIGHTS PRELIMS TEST SERIES 2017

www.insightsonindia.com

TEST 1 Solutions

www.insightsias.com

The FATF's decision making body, the FATF Plenary, meets three
times per year.
Q Source: http://www.thehindu.com/news/national/india-seeks-us-helpto-crack-pakistan-terror-fund-trail/article8961156.ece


29.
Which of the following forest types is common to Andaman and
Nicobar Islands, parts of North-Eastern states and a narrow strip of
the Western slope of the Western Ghats?
a) Mangrove forests
b) Temperate forests
c) Tropical moist forests
d) Coral forests

Solution: c)
Justification: Option (a): Mangroves are found in A&N islands and even
near the western coast of India, but not in NE states, or on the Western
slope of Western Ghats.
Option (b): Temperate forests are found in temperate latitudes or at higher
altitudes. A&N islands do not host temperate forests.
Option (d): Corals are found largely in A&N, Lakshadweep, Gulf of Mannar
(near TN) and Gulf of Kutch.
Option (c): Tropical moist forests include evergreen forests; Tropical SemiEvergreen Forests, Tropical Moist Deciduous Forests and Littoral and
Swamp Forests.
The Middle Andamans harbours mostly moist deciduous forests.
North Andamans is characterised by the wet evergreen type.
Western Ghats and NE states harbour both deciduous and evergreen
forests.
We will be covering more questions specific to vegetation in various
parts of India in later tests.

INSIGHTS PRELIMS TEST SERIES 2017

www.insightsonindia.com

TEST 1 Solutions

www.insightsias.com

Q Source: Page 58: NCERT 6th Geography: The Earth Our Habitat




30.
Jupiter, Saturn and Uranus have rings around them. These
rings consist of
a) Satellites of these planets only
b) Icy dust, large rocks and moons of the planets
c) Comets and asteroids only
d) Magnetospheric flux of the planet

Solution: b)
Learning: The composition of ring particles varies; they may be silicate or
icy dust. Larger rocks and boulders may also be present. Sometimes rings
will have small moons that orbit near the inner or outer edges of rings or
within gaps in the rings.\
For e.g. the ring swirling around Saturn consists of chunks of ice and dust.
Recent evidence suggests that ring systems may be found around other
types of astronomical objects, including minor planets, moons, and brown
dwarfs.
Q Source: Page 2 (Side Box): NCERT 6th Geography: The Earth Our Habitat



31.
Consider the following species and the regions where they are
found in.

Species

Region

A. Wild Ass

1. Assam

INSIGHTS PRELIMS TEST SERIES 2017

www.insightsonindia.com

TEST 1 Solutions

B. Wild Goat

C. One-horned Rhinoceros

www.insightsias.com

2. Runn of Kutch
3. Himalayan region

The correct match for the above will be?


a)
b)
c)
d)

A-1, B-3, C-2


A-2, B-3, C-1
A-3, B-1, C-2
A-2, B-1, C-3

Solution: b)
Justification: Elephants and one-horned rhinoceroses (categorized
vulnerable as per IUCN) are found in the forests of Assam, especially
Kaziranga national park.
Camels and wild asses are found in the Great Indian desert and the Rann of
Kuchchh, which is also the largest wildlife sanctuary in India. As of 2016,
the Indian wild ass is listed as Near Threatened by IUCN.
Wild goats, snow leopards, bears, etc. are found in the Himalayan region.
Some of these wild goats are reared for the famous Pashmina wool.
Q Source: Page 63: NCERT 6th Geography: The Earth Our Habitat



32.
The Future We Want document was a result of which of the
following conferences?
a) Earth Summit 2012 at Rio
b) Warsaw Climate Change Agreement
c) Sustainable Development Goals (SDGs) meeting at UN General
Assembly
d) The First UN conference on Sustainable development

Solution: a)

INSIGHTS PRELIMS TEST SERIES 2017

www.insightsonindia.com

TEST 1 Solutions

www.insightsias.com

Learning: It was a nonbinding document, "The Future We Want," a work


paper.
In it, the heads of state of the 192 governments in attendance renewed
their political commitment to sustainable development and declared their
commitment to the promotion of a sustainable future.
The document largely reaffirms previous action plans like Agenda 21.
Q Source: https://sustainabledevelopment.un.org/rio20/futurewewant




33.
Consider the following statements about the historical
inscriptions in India.
1. All the edicts of Asoka engraved in the Kharoshthi and Brahmi
scripts are in the Prakrit language.
2. With the rise of the Guptas, Prakrit became the predominant
language of Indian epigraphs.
Which of the above is/are correct?
a)
b)
c)
d)

1 only
2 only
Both 1 and 2
None

Solution: a)
Justification: Statement 1: This is with reference to Epigraphical Studies in
India - Sanskrit and Dravidian Language of the inscriptions.
We are quoting verbatim from ASIs website, The language of the
earliest written records, viz., the Asokan edicts, is Prakrit. Besides
Prakrit, Asokan edicts are written in Greek and Aramaic languages

INSIGHTS PRELIMS TEST SERIES 2017

www.insightsonindia.com

TEST 1 Solutions

www.insightsias.com

also. All the edicts of Asoka engraved in the Kharoshthi and Brahmi
scripts are in the Prakrit language.
Thus, originally the language employed in the inscriptions was
Prakrit and Sanskrit was adopted in the inscriptions only at a later
period.
Statement 2: After the period of Asoka, the use of the Prakrit language
continued in inscriptions for a few more centuries.
In north India, Prakrit was replaced by Sanskrit about the end of 3rd
century A.D. while this change took place about a century later in
south India.
From the 4th century onwards, with the rise of the Guptas, Sanskrit
became the predominant language of Indian epigraphs. So, 2 is
wrong.
This is because Guptas were great patrons of Brahmanism and
Sanskrit language.
Learning: The contemporary rulers of the Guptas, in central India and
parts of the Deccan, like the Vakatakas, the Kadambas and later Gangas of
Karnataka and the Pallavas in south India also employed Sanskrit in their
inscriptions.
The copper-plate charters of the Pallavas, the Cholas and the Pandyas are
written in both Sanskrit and Tamil languages.
Q Source: Improvisation: Page 5: NCERT 6th History: Our Pasts-I




34.
Consider the following about the Rock Shelters of Bhimbetka.
1. Bhimbetka derives its name from the Atharva Veda.
2. Mesolithic as well as Medieval period rock paintings can be found
in the caves.
3. It is a UNESCO World Heritage Site.

INSIGHTS PRELIMS TEST SERIES 2017

www.insightsonindia.com

TEST 1 Solutions

www.insightsias.com

4. Bhimbetka paintings are highly secular in nature.


Select the correct answer using the codes below.
a)
b)
c)
d)

2 and 4 only
1 and 3 only
2 and 3 only
1 and 4 only

Solution: c)
Justification: Statement 1: Bhimbetka owes its name to the characters of
the longest epic in the world, the Mahabharata. It is believed that when the
five brothers, called Pandavas, were banished from their kingdom, they
came here and stayed in these caves, the massive rocks seating the gigantic
frame of Bhima, the second Pandava.
Statement 2: Bhimbetka is an archaeological treasure. The rock paintings
have numerous layers belonging to various epochs of time, ranging from
the Upper Palaeolithic, Mesolithic Age to the protohistoric, early historic
and medieval periods. The most ancient scenes here believed to be
commonly belonging to the Mesolithic Age. So, 2 is correct.
Statement 3: You can read more about it here
http://asi.nic.in/asi_monu_whs_rockart_bhimbetka_detail.asp
Statement 4: Animals such as bison, tiger, rhinoceros, wild boar, elephants,
monkeys, antelopes, lizards, peacocks etc. have been abundantly depicted
in the rock shelters. Popular religious and ritual symbols also occur
frequently. So, 4 is false.
Q Source: Improvisation: Page 14-15: NCERT 6th History: Our Pasts-I



35.
Mikir hills, Mishmi hills and Barail range are located in
a) North-eastern India
b) Southern India

INSIGHTS PRELIMS TEST SERIES 2017

www.insightsonindia.com

TEST 1 Solutions

www.insightsias.com

c) Northern Ladakh
d) Western Aravallis

Solution: a)
Learning: These hills are famous for their tribal inhabitants. We will be
covering specific tribes of these regions in later tests.
Note the order of the hills and the state they are located in.


Q Source: Map-based questions : India


36.
Consider the following statements about the India Nuclear
Insurance Pool (INIP).
1. It was launched as per the mandatory provision under the Civil
Liability for Nuclear Damage Act (CLND), 2010.
2. The pool provides capacity for insurance coverage to citizens
affected by nuclear hazards in India.
3. It will fund future development of those nuclear reactors which
involve considerable risks due to geology or radiation.
Select the correct answer using the codes below.
a) 1 only

INSIGHTS PRELIMS TEST SERIES 2017

www.insightsonindia.com

TEST 1 Solutions

www.insightsias.com

b) 2 and 3 only
c) 1 and 2 only
d) 1, 2 and 3

Solution: a)
Justification: Statement 1: Based on Obamas visit, a INIP was negotiated,
which then came into effect via the CLND Act.
Statement 2: The pool provides capacity for insurance coverage to
operators and suppliers for any nuclear liability towards third party. INIP
also will offer policies on the nuclear operators liability insurance policy
and a nuclear suppliers special contingency (against right to recourse)
insurance policy.
Statement 3: There is no such provision. It is an insurance pool only, not a
development pool.
Q Source: http://www.thehindu.com/business/nuclear-suppliersinsurance-policy-unveiled/article8981843.ece



37.
Consider the following about the S.A.M.O.A. Pathway.
1. It is a part of the Paris Climate Change Agreement 2015.
2. It focuses on those communities in developing countries that
derive their livelihood exclusively from forestry and use of other
natural resources.
Which of the above is/are correct?
a)
b)
c)
d)

1 only
2 only
Both 1 and 2
None

Solution: d)

INSIGHTS PRELIMS TEST SERIES 2017

www.insightsonindia.com

TEST 1 Solutions

www.insightsias.com

Justification: SAMOA was an outcome document of the Third


International Conference on Small Island Developing States (SIDS), the
'SIDS Accelerated Modalities of Action (SAMOA) Pathway.
The Samoa Pathway recognizes the adverse impacts of climate change and
sea-level rise on SIDS efforts to achieve sustainable development as well as
to their survival and viability, and addresses economic development, food
security, disaster risk reduction (DRR) and ocean management, among
other issues.
Since the agreement came about in 2014, it cannot be a part of the Paris
Climate agreement 2015. So, 1 is wrong.
Q Source: Last 2 years current affairs:
https://sustainabledevelopment.un.org/sids2014/samoapathway



38.

Consider the following.

Communities seen in news

Belong to

1. Tuaregs

A. Sahara Desert

2. Sinhalas

B. Bhutan

3. Rakhine

C. Sri Lanka

Which of these is/are correctly matched?


a)
b)
c)
d)

2 and 3
1 only
1 and 3 only
1 and 2 only

Solution: b)
Justification: Statement 1: These are people with a nomadic pastoralist
lifestyle. They are the principal inhabitants of the vast Sahara Desert.

INSIGHTS PRELIMS TEST SERIES 2017

www.insightsonindia.com

TEST 1 Solutions

www.insightsias.com

They raged the Tuareg Rebellion (2012) which was an early stage of the
Northern Mali conflict waged against the Malian government. Their goal
was attaining independence for the northern region of Mali, known as
Azawad.
Statement 2: Sinhalas are the major community of Sri Lanka often in
conflict with the Tamils of SL.
Statement 3: Rakhine is a state in Myanmar, and also a principal ethnic
community. The 2012 Rakhine State riots were a series of conflicts
primarily between ethnic Rakhine Buddhists and Rohingya Muslims in
northern Rakhine State.
Q Source: Often in news + Improvisation: Page 73: Geography NCERT 7th:
Our Environment




39.
Consider the following about the Malware Infection Index 2016
(MII 2016) for Asia-Pacific region.
1. It was released by Internet Corporation for Assigned Names and
Numbers (ICANN).
2. The Index pin points India to be the most severely affected
country in the entire region.
Which of the above is/are correct?
a)
b)
c)
d)

1 only
2 only
Both 1 and 2
None

Solution: d)
Justification: Statement 1: Microsoft Asia announced the launch of its
Malware Infection Index 2016 (MII2016), which identifies the key malware

INSIGHTS PRELIMS TEST SERIES 2017

www.insightsonindia.com

TEST 1 Solutions

www.insightsias.com

threats in the region and ranks markets in Asia Pacific according to how
much they are affected.
The region is especially vulnerable with emerging markets most at risk of
malware threats.
Statement 2: The image below clearly shows that countries like Pakistan
and some ASEAN nations are most affected. India is less severely affected.


Q Source: http://timesofindia.indiatimes.com/tech/tech-news/Pakistantops-the-malware-index-list-India-8thMicrosoft/articleshow/52636322.cms

INSIGHTS PRELIMS TEST SERIES 2017

www.insightsonindia.com

TEST 1 Solutions

www.insightsias.com




40.
a)
b)
c)
d)

Einstein Ring can be observed when


All planets in the solar system fall in a common line
A pair of distant galaxies from earth aligns in a special position
with respect to earth
Solar eclipse is just about to end
Volcanic eruptions occur at the craters of the Jovian planets

Solution: b)
Learning: Einstein Ring was discovered recently by an Institute of Chile.
"Einstein Ring", first predicted by Einstein's theory of General
Relativity, is a rarely observed, astronomical phenomenon that
occurs when two distant galaxies are perfectly aligned, millions of
light years apart.
Because the two galaxies are aligned so perfectly, light from the most
distant, or source galaxy, is being bent by the gravity of the closer
one.
This makes the light from the furthest galaxy appear as an almost
perfect circle when viewed from Earth.
An Einstein ring is a distorted image of a very distant galaxy, which is
termed 'the source'.
Q Source: http://www.space.com/33095-nearly-perfect-einstein-ringdiscovered-image.html


INSIGHTS PRELIMS TEST SERIES 2017

www.insightsonindia.com

TEST 1 Solutions

www.insightsias.com

41.
This ruler convened a Buddhist Council at Kashmir which
eventually led to the recognition of the Mahayana sect of Buddhism.
He is?
a) Ashoka
b) Yavana
c) Kanishka
d) Kumaragupta

Solution: c)
Justification & Learning: The Fourth Buddhist Council was held at
Kundalvana, Kashmir in 72 AD under the patronage of Kushan king
Kanishka.
The president of this council was Vasumitra.
This council distinctly divided the Buddhism into 2 sects Mahayana
and Hinayana.
Another Fourth Buddhist Council was held at Tambapanni (one name
of Sri Lanka) at Aloka Lena under the patronage of VattagamaniAbaya. This was Theravada Buddhist council.
Q Source: Page 103: NCERT 6th History: Our Pasts-I



42.
Consider the following statements with reference to some of
the officials in the History of India.
1. Nagara-shreshthi was the Chief Craftsman of the royal court.
2. Sarthavaha was the leader of bonded labourers.
3. Sandhi-vigrahika was a minister of war and peace.
Select the correct answer using the codes below.
a)
b)
c)
d)

1 and 2 only
2 and 3 only
3 only
None of the above

INSIGHTS PRELIMS TEST SERIES 2017

www.insightsonindia.com

TEST 1 Solutions

www.insightsias.com

Solution: c)
Justification & Learning: Kings adopted a number of steps to win the
support of men who were powerful, either economically, or socially, or
because of their political and military strength.
Statement 1 and 2: Important men probably had a say in local
administration. These included the nagarashreshthi or chief banker or
merchant of the city, the sarthavaha or leader of the merchant caravans, the
prathama-kulika or the chief craftsman, and the head of the kayasthas or
scribes.
Statement 3: Sometimes, one person held many offices. For instance,
besides being a maha-danda-nayaka, Harishena was a kumar-amatya,
meaning an important minister, and a sandhi-vigrahika, meaning a minister
of war and peace.
Q Source: Page 116: NCERT 6th History: Our Pasts-I



43.
Consider the following about National Mineral Exploration
Trust (MET).
1. It grants mining leases for major minerals to after obtaining
sanction from the State government.
2. It carries out detailed exploration for minerals using the funds
accrued to it.
3. It invests in basic infrastructural development and implements
welfare schemes in mining affected belts.
Select the correct answer using the codes below.
a)
b)
c)
d)

1 only
2 only
1 and 3 only
1 and 2 only

INSIGHTS PRELIMS TEST SERIES 2017

www.insightsonindia.com

TEST 1 Solutions

www.insightsias.com

Solution: b)
Justification: Statement 1: Major minerals are administered by the Central
government. Moreover, this body doesnt give sanction for mining leases.
So, 1 is wrong.
Statement 2: The Mines and Minerals (Development & Regulation)
Amendment Act, 2015, (MMRDA) mandated the setting up of Mineral
Exploration Trust.
It is a non-profit body by the Central Government for the purposes of
regional and detailed exploration of minerals using the funds accrued to it
and in such manner as prescribed by the Central Government.
Statement 3: It is done by District Mineral Foundation. So, 3 is wrong.
Q Source: Other details can be seen here http://www.businessstandard.com/article/pti-stories/govt-notifies-norms-for-nationalmineral-exploration-trust-115082100414_1.html



44.
a)
b)
c)
d)

Atmosphere is held around the earth due to


Continuous magnetic activity in the Earths core
Corona discharge from the Sun that reaches the earth
consistently
Gravitational pull of the earth
Presence of clouds in the troposphere and other layers above it

Solution: c)
Justification: Celestial bodies like Moon dont have an atmosphere because
their gravity isnt enough to hold the atmosphere together with the body.
Since gravity is low, and gases are lighter, they escape from the celestial
body.

INSIGHTS PRELIMS TEST SERIES 2017

www.insightsonindia.com

TEST 1 Solutions

www.insightsias.com

This is not the case with earth, where gravitation is high enough to hold the
gases together. Escape velocity of gases on earth is much higher than that
on some other celestial bodies like Moon.
Learning: Atmosphere also protects us from the harmful rays and
scorching heat of the sun. It consists of a number of gases, dust and water
vapour.
The changes in the atmosphere produce changes in the weather and
climate.
Q Source: Page 3: Geography NCERT 7th: Our Environment



45.
a)
b)
c)
d)

Ecosystem is formed, most appropriately, by the interaction of


Both biotic and abiotic factors of the environment
All Living organisms present in the system
Diverse Landforms found within a large zone
Communities found near edge of a biome

Solution: a)
Justification: An ecosystem is a system formed by the interaction of all
living organisms (biotic) with each other and with the physical and
chemical factors (abiotic) of the environment in which they live, all linked
by transfer of energy and material.
Biome is a larger region than ecosystem, and comprises many such
ecosystems. So, D is incorrect.
We will be covering more questions on the hierarchies of ecology.
Q Source: Page 5: Geography NCERT 7th: Our Environment

INSIGHTS PRELIMS TEST SERIES 2017

www.insightsonindia.com

TEST 1 Solutions

www.insightsias.com


46.
Consider the following about state administration.
1. Governor of the State is appointed by the Central Government but
the State Assembly may disapprove her appointment in effect
repealing it.
2. Governor of the state appoints the chief minister and other
ministers only after obtaining the advice of the Central Council of
Ministers.
Which of the above is/are correct?
a)
b)
c)
d)

1 only
2 only
Both 1 and 2
None

Solution: d)
Justification: Statement 1: Governor is appointed by the President. State
has no say in either the appointment or removal of the Governor. So, 1 is
wrong.
Statement 2: Governor doesnt consult the Union government in making
state appointments, even though the Governor is an agent of the Centre.
She is bound to choose the leader of the largest party/coalition as the CM
and others recommended by the CM as ministers.
Q Source: Improvisation: Page 33: NCERT 7th: Social and Economic Life II





INSIGHTS PRELIMS TEST SERIES 2017

www.insightsonindia.com

TEST 1 Solutions

www.insightsias.com

47.
How the National Highways Grid proposed by the NHAI will be
useful?
1. The Grid will improve connectivity between coastal areas and
state capitals thus facilitating cargo movement.
2. It will help the government re-designate the National Highways
for easy identification.
Which of the above is/are correct?
a)
b)
c)
d)

1 only
2 only
Both 1 and 2
None

Solution: c)
Justification: Statement 1: The grid will connect 12 major ports, 26 state
capitals and more than 45 cities and thus help in quick evacuation and
transport of cargo from one end to the other.
Statement 2: In India the lack of scientific road network pattern has
created a problem for drivers who can't take a straight road to reach from
one place to the other.
National Highway Authority of India is proposing a National
Highway Grid which will include 27 horizontal and vertical highway
corridors spread across the country.
The corridors, spaced at a distance of 250 kilometres, will crisscross
and connect with each other.
The preparation of the grids will help the government re-designate
the NHs for easy identification. For example, all even number NH
grids can be identified for roads connecting east to west and odd
number for corridors joining north end to south
Q Source:
http://economictimes.indiatimes.com/news/economy/infrastructure/gov
ernment-plans-national-highway-grids-for-smooth-travel-27-corridorsidentified/articleshow/52723683.cms

INSIGHTS PRELIMS TEST SERIES 2017

www.insightsonindia.com

TEST 1 Solutions

www.insightsias.com




48.
Greenhouse gases that are present in the atmosphere due to
NATURAL activities is/are?
1. Chlorofluorocarbons
2. Methane
3. Nitrous oxide
4. Sulfur hexafluoride
5. Oxygen
6. Water vapour
Select the correct answer using the codes below.
a)
b)
c)
d)

2, 4 and 5 only
2, 3 and 6 only
1, 3, 4, 5 and 6 only
1, 2 and 6 only

Solution: b)
Justification: Statement 1: CFC is a manmade gas, and so all options
containing CFC will be wrong.
Statement 2: Methane is emitted by rice field, animals (digestive tracts),
coal mines etc.
Statement 3: Nitrous oxide (N2O) is produced by fertilizer use, animal
waste management, fossil fuel combustion, industrial activities, soils under
natural vegetation and the oceans. So, it is produced by both natural and
human activities. 3 will still be correct.
Statement 4: SF6 is an inorganic extremely potent greenhouse gas, which is
an excellent electrical insulator. It is a manmade gas.
Statement 5: Oxygen (O2) is not a GHG, ozone (O3) is.
Statement 6: Water vapour is a very important GHG, and it is obviously
natural.

INSIGHTS PRELIMS TEST SERIES 2017

www.insightsonindia.com

TEST 1 Solutions

www.insightsias.com

Q Source: Improvisation: Page 20: Geography NCERT 7th: Our Environment





49.
Optically-Stimulated Luminescence is a dating technique that is
used to find out
a) The last time quartz or feldspar sediment was exposed to light
b) Radiation emitted by antecedent water bodies
c) Age of tree barks by exposing them to ionizing radiation
d) Duration of photons incident on any surface

Solution: a)
Learning: Indian researchers have used dating techniques on animal
remains and pottery fragments to conclude that the Indus Valley
settlements could be older than previously believed.
They also used the technique called Optically-Stimulated Luminescence.
It is a late Quaternary dating technique used to date the last time
quartz sediment was exposed to light.
As sediment is transported by wind, water, or ice, it is exposed to
sunlight and zeroed of any previous luminescence signal.
Once this sediment is deposited and subsequently buried, it is
removed from light and is exposed to low levels of natural radiation
in the surrounding sediment.
The technique measures their age using ionizing radiation.
You can read more here http://timesofindia.indiatimes.com/india/Indusera-8000-years-old-not-5500-ended-because-of-weakermonsoon/articleshow/52485332.cms
Q Source: As given above

INSIGHTS PRELIMS TEST SERIES 2017

www.insightsonindia.com

TEST 1 Solutions

www.insightsias.com



50.
Temperature in cities is much higher than that of villages due
to
1. Lower forest coverage in cities as compared to villages
2. Concrete and asphalt structure in cities capture and release heat
3. Vehicular emissions in cities release Greenhouse gases that trap
heat
4. Cities are more densely populated as compared to villages
5. The structure of the economic activity of cities
Select the correct answer using the codes below.
a)
b)
c)
d)

3, 4 and 5 only
1, 2 and 3 only
2, 4 and 5 only
1, 2, 3, 4 and 5

Solution: d)
Justification: Statement 1: Forest moderate temperature, cool the
surroundings by transpiration and capture the carbondioxide that traps
heat. Deforestation and low forest cover contributes to urban heat islands.
Statement 2: The concrete and metals in buildings and the asaphalt of
roads get heated up during the day. This heat is released during the night.
Also, the crowded high rise buildings of the cities trap the warm air and
thus raise the temperature of the cities.
Statement 3: Vehicle exhaust emits CO2, NOX and other GHGs that capture
heat.
Statement 4 and 5: Density of population and industrial economic activity
automatically increase the heat generated per unit area.
Q Source: Improvisation: Page 24: Geography NCERT 7th: Our Environment

INSIGHTS PRELIMS TEST SERIES 2017

www.insightsonindia.com

TEST 1 Solutions

www.insightsias.com




51.
of

Direct Heat of the Sun can be felt immensely at Ladakh because


a)
b)
c)
d)

High water vapour saturation in air


Day temperatures are very high in Ladakh
Thin air at Ladakh
High albedo of ice as compared to regions down the hill

Solution: c)
Justification: The altitude in Ladakh varies from about 3000m (in Kargil)
to more than 8,000m (in the Karakoram).
Due to such a high altitude, the climate is extremely cold and dry. So,
option A and B are wrong.
Option D is also wrong because albedo of Ice reflects the sunlight,
whereas here we are talking about direct rays of the Sun.
The air at this altitude is so thin that the heat of the sun can be felt
intensely.
The area experiences freezing winds and burning hot sunlight.
You will be surprised to know that if you sit in the sun will your feet
in the shade, you may suffer from both sunstroke and frost bite at the
same time.
Learning: The day temperatures in summer are just above zero degree and
the night temperatures well below 30C. It is freezing cold in the winters
when the temperatures may remain below 40C for most of the time. As it
lies in the rain shadow of the Himalayas, there is little rainfall, as low as 10
cm every year.
Q Source: Improvisation: Page 74: Geography NCERT 7th: Our Environment

INSIGHTS PRELIMS TEST SERIES 2017

www.insightsonindia.com

TEST 1 Solutions

www.insightsias.com



52.
Kuji Mendha is a breed of sheep found only in coastal Odisha.
Why it was recently conferred rare and singular species tag by the
Union government?
a) It was declared Near Extinction in the IUCN Red List.
b) It has some rare genetic traits which lead to multiple birth
syndromes in it.
c) It survives in the harshest conditions known to the breed of
this species and still highly productive in terms of wool and
meat.
d) All of the above

Solution: b)
Justification: Scientific studies on this rare breed found the sheep to be
carrying a rare gene mutation.
Kuji Mendha sheep are fast breeders giving multiple birth.
Sheep in other parts of Odisha are not known for giving multiple
birth. This characteristic makes them distinctive from other species.
Learning: They are well adapted to high ambient temperature, high
humidity and heavy rains.
They are found in Jagatsinghpur and Kendrapara districts of Odisha.
The National Bureau of Animal Genetic Resources (NBAGR) has
accorded the genetic recognition.
With NBAGR conferring it genetically rare status, conserving these
domesticated species would receive a boost.
Q Source: http://timesofindia.indiatimes.com/home/environment/florafauna/Kendrapara-sheep-gets-rare-status/articleshow/53140281.cms

INSIGHTS PRELIMS TEST SERIES 2017

www.insightsonindia.com

TEST 1 Solutions

www.insightsias.com


53.
1.
2.
3.
4.
5.

Consider the following sources of freshwater on earth.


Groundwater
Atmosphere
Ice caps
Fresh water lakes
Rivers

Arrange the following in increasing order of availability of freshwater from


the codes given below.
a)
b)
c)
d)

2<1<4<5<3
2<4<1<3<5
1<2<4<5<3
5<2<4<1<3

Solution: d)
Justification: You should first apply elimination in such questions. It is
common knowledge that ice-caps are the largest source of fresh water,
which eliminates B.
Also, since groundwater is more abundant than river water, it eliminates C
and A.
The clear answer is D.
Distribution of water as given in NCERT.

INSIGHTS PRELIMS TEST SERIES 2017

www.insightsonindia.com

TEST 1 Solutions

www.insightsias.com

It can be understood by an analogy.


Q Source: Page 32: Geography NCERT 7th: Our Environment


54.
The Ministry of Environment, Forest and Climate Change
(MoEFCC) has suggested that it will propose for the funding of foam
industries to shift to cyclopentane. How the shift to cyclopentane will
be useful?
1. It has zero ozone depletion potential and low global warming
potential.
2. It has very low flammability and will help avoid fire hazards in
industries.
Which of the above is/are correct?
a)
b)
c)
d)

1 only
2 only
Both 1 and 2
None

Solution: a)

INSIGHTS PRELIMS TEST SERIES 2017

www.insightsonindia.com

TEST 1 Solutions

www.insightsias.com

Justification: Statement 1: The shift will be made from


Hydrochlorofluorocarbons (HCFCs) which are ozone depleting.
HCFC is a commonly used refrigerant gas. It is also used in foam
blowing agents, solvents, aerosols and fire extinguishers.
India is in the process of phasing out HCFC by 2030 as per the
existing agreement signed by India in the Montreal protocol.
The transition will be funded by the Executive Committee of the
Multilateral fund as per the Montreal Protocol Agreement in the
second stage of HCFC phase out. SMEs will encouraged to shift to
cyclopentanes.
Statement 2: Cyclopentane is a cheaper alternative (HCFC is patented and
expensive) without any patents associated with it.
But, some SMEs also expressed their reservations against using
cyclopentane due to its flammability and the need for skilled technicians to
handle it.
Q Source: http://www.downtoearth.org.in/news/india-decides-to-put-abreak-on-hfo-in-foam-sector-for-now-55227


55.
Consider the following about the Directorate of Enforcement.
1. It is housed under the Ministry of Corporate Affairs.
2. It enforces the Prevention of Money Laundering Act.
3. It keeps track of foreign exchange movement in and out of India.
Select the correct answer using the codes below.
a)
b)
c)
d)

1 and 2 only
2 and 3 only
1 only
2 only

Solution: b)

INSIGHTS PRELIMS TEST SERIES 2017

www.insightsonindia.com

TEST 1 Solutions

www.insightsias.com

Justification: Statement 1: Directorate of Enforcement is a specialized


financial investigation agency under the Department of Revenue, Ministry
of Finance. So, 1 is wrong.
Statement 2 and 3: It enforces the Foreign Exchange Management Act,1999
(FEMA) and Prevention of Money Laundering Act, 2002 (PMLA).
The Act and Rules notified there under impose obligation on banking
companies, financial institutions and intermediaries to verify identity of
clients, maintain records and furnish information
You can read more about the laws here at the Q Source.
Q Source: http://www.insightsonindia.com/2016/08/02/insights-dailycurrent-affairs-02-august-2016/



56.
in

Impeyan monal is most likely to be found in its natural habitat


a)
b)
c)
d)

Adjoining oases of the hot deserts of Rajasthan


Swamp forests of Andaman & Nicobar Islands
Temperate forests in the Himalayan region
Salt marshes of Gujarat

Solution: c)
Learning: Himalayan monal is also known as the Impeyan monal or
pheasant is state bird of Uttarakhand and national bird of Nepal, where it is
known as Danphe. It was also the state bird of Himachal Pradesh, until
2007.
The bird's natural range extends from eastern Afghanistan through
the Himalayas in Pakistan, Kashmir region and the Republic of India
(states of Himachal Pradesh, Uttarakhand, Sikkim and Arunachal
Pradesh), Nepal, southern Tibet, and Bhutan.

INSIGHTS PRELIMS TEST SERIES 2017

www.insightsonindia.com

TEST 1 Solutions

www.insightsias.com

It occupies upper temperate oak-conifer forests interspersed with


open grassy slopes, cliffs and alpine meadows
In some areas, the species is threatened due to poaching and other
anthropogenic factors.


Q Source: Improvisation: Page 42: Fig 6.10: Geography NCERT 7th: Our
Environment




57.
Which of the following differentiates correctly between
Tropical Savannahs and Temperate Grasslands?
a) Temperate grass is longer but much less nutritious than those
of Tropical Savannas.
b) Temperate grasslands are drier and receive lesser rainfall than
savannas.

INSIGHTS PRELIMS TEST SERIES 2017

www.insightsonindia.com

TEST 1 Solutions

www.insightsias.com

c) Temperate Grasslands have scattered trees, unlike in Savannas


where the only vegetation is grass and mosses.
d) Temperate grasslands are found near the coastal regions,
whereas Tropical Savannas occur in interior areas in Northern
Hemisphere only.

Solution: b)
Justification: Option A: Temperate grass here is short and nutritious, not
longer than Savannah. So, A is wrong.
Option B & C: Savannah is Grassland characterized by scattered trees that
are not able to form a canopy. As there is no canopy, the surface gets
sufficient light, and the area supports grasses.
Temperate grasslands are characterized by grasses of different types with
no trees and only a few types of shrubs. The reason of no trees lies in the
fact that temperate grasslands receive lesser rainfall and are drier. So, C is
wrong and B is correct.
Option D: These grow on either side of the equator and extend till the
tropics. This vegetation grows in the areas of moderate to low amount of
rainfall.
Temperate Grasslands are found in the mid-latitudinal zones and in the
interior part of the continents. So, D is wrong.
Q Source: Improvisation: Page 43: Geography NCERT 7th: Our Environment



58.
Apart from Wheat, Crops that are usually grown in the plains of
India are
1. Maize
2. Sorghum
3. Coffee
4. Millets

INSIGHTS PRELIMS TEST SERIES 2017

www.insightsonindia.com

TEST 1 Solutions

www.insightsias.com

5. Sugarcane
Select the correct answer using the codes below.
a)
b)
c)
d)

1 and 5 only
1, 2, 4 and 5 only
2, 3 and 4 only
1, 2, 3, 4 and 5

Solution: b)
Justification: The soil can be fertile in plain areas. This is ideal for growing
crops like Sugarcane, Jute, sorghum, rice, wheat etc.
Millets can even grow in relatively drier areas with less fertile soils.
Banana plantations are seen in some areas of the plain.
Coffee and tea are generally grown in hilly areas. So, 3 is wrong.
Q Source: Page 60: Geography NCERT 7th: Our Environment



59.
The Buddha taught that human life is full of suffering and
unhappiness, which is caused due to
a) Separation of matter with soul
b) Law of Karma
c) Constant cravings and desires
d) Following the middle way in everything which dilutes focus
and energy

Solution: c)
Justification: The suffering is caused because we have cravings and desires
(which often cannot be fulfilled).

INSIGHTS PRELIMS TEST SERIES 2017

www.insightsonindia.com

TEST 1 Solutions

www.insightsias.com

Sometimes, even if we get what we want, we are not satisfied, and


want even more (or want other things). The Buddha described this as
thirst or tanha.
He taught that this constant craving could be removed by following
moderation in everything. So, clearly (d) is wrong.
Buddha doesnt pin point the law of karma to be responsible for our
suffering. He only believed that the results of our actions (called
karma), whether good or bad, affect us both in this life and the next
(some authorities deny this aspect of Buddhism). So, (b) cant be
correct, if option (c) is already mentioned as later is more
appropriate.
Learning: He also taught people to be kind, and to respect the lives of
others, including animals.
The Buddha taught in the language of the ordinary people, Prakrit, so that
everybody could understand his message.
Q Source: Improvisation: Page 66: NCERT 6th History: Our Pasts-I



60.
1.
2.
3.
4.

Consider the following locations in Northern India.


Dehra Gompas
Siachen Glacier
Rohtang pass
Zoji la pass

The correct order of these regions from North to South is?


a)
b)
c)
d)

2143
2314
1342
3241

Solution: a)

INSIGHTS PRELIMS TEST SERIES 2017

www.insightsonindia.com

TEST 1 Solutions

www.insightsias.com

Justification: You should use elimination.


You must have heard of the Manali to Rohtang pass way. On the other hand,
Siachen Glacier is known be located in Northern J&K. This implies that
Rohtang pass (statement 3) must lie to the South of Siachen Glacier
(statement 2).
So, C and D can be easily eliminated.
If you know that Zoji la pass lies between Leh and Srinagar, it implies that
Zoji la must lie to the North of Rohtang pass. So, the only answer possible is
option A.
Learning: Manali - Leh highway crosses four passes, Rohtang la, Baralacha
la Lungalacha la and Tanglang la.


Q Source: Fig 10.4: Page 74: Geography NCERT 7th: Our Environment

INSIGHTS PRELIMS TEST SERIES 2017

www.insightsonindia.com

TEST 1 Solutions

www.insightsias.com




61.
Consider the following statements.
1. The air pressure is highest at sea level and decreases with height.
2. Low air pressure is generally associated with cloudy skies and wet
weather.
Which of the above is/are correct?
a)
b)
c)
d)

1 only
2 only
Both 1 and 2
None

Solution: c)
Justification: Statement 1: Air pressure is defined as the pressure exerted
by the weight of air on the earths surface. As we go up the layers of
atmosphere, the pressure falls rapidly.
Statement 2: In areas where temperature is high the air gets heated and
rises. This creates a low-pressure area. Low pressure is associated with
cloudy skies and wet weather.
In areas having lower temperature, the air is cold. It is therefore
heavy.
Heavy air sinks and creates a high pressure area. High pressure is
associated with clear and sunny skies.
The air always moves from high pressure areas to low pressure
areas.
Q Source: Page 24: Geography NCERT 7th: Our Environment

INSIGHTS PRELIMS TEST SERIES 2017

www.insightsonindia.com

TEST 1 Solutions

www.insightsias.com



62.
1.
2.
3.
4.

Factors that affect the formation of tides in the Ocean are


Alignment of Earth, Sun and Moon
Relative distance between the Moon, Earth and Sun
Shape of bays and estuaries where tides are formed
Local wind and weather patterns in Ocean

Select the correct answer using the codes below.


a)
b)
c)
d)

1 and 2 only
2 and 3 only
1 and 4 only
1, 2, 3 and 4

Solution: d)
Justification: Statement 1 and 2: The strong gravitational pull exerted by
the sun and the moon on the earths surface causes the tides.
The water of the earth closer to the moon gets pulled under the influence of
the moons gravitational force and causes high tide.
During the full moon and new moon days, the sun, the moon and the
earth are in the same line and the tides are highest. These tides are
called spring tides.
But when the moon is in its first and last quarter, the ocean waters
get drawn in diagonally opposite directions by the gravitational pull
of sun and earth resulting in low tides. These tides are called neap
tides.
Statement 3: The shape of bays and estuaries also can magnify the
intensity of tides. Funnel-shaped bays in particular can dramatically alter
tidal magnitude.
The Bay of Fundy in Nova Scotia is the classic example of this effect,
and has the highest tides in the world - over 15 meters.

INSIGHTS PRELIMS TEST SERIES 2017

www.insightsonindia.com

TEST 1 Solutions

www.insightsias.com

Narrow inlets and shallow water also tend to dissipate incoming


tides.
In estuaries with strong tidal rivers, such as the Delaware River and
Columbia River, powerful seasonal river flows in the spring can
severely alter or mask the incoming tide.
Statement 4: Strong offshore winds can move water away from coastlines,
exaggerating low tide exposures.
Onshore winds may act to pile up water onto the shoreline, virtually
eliminating low tide exposures.
High-pressure systems can depress sea levels, leading to clear sunny
days with exceptionally low tides.
Conversely, low-pressure systems that contribute to cloudy, rainy
conditions typically are associated with tides than are much higher
than predicted.
Q Source: Improvisation: Page 36: Geography NCERT 7th: Our Environment



63.
a)
b)
c)
d)

Anantnag, Baramula, and Pahalgam are well known for


Production of Kashmiri willow Cricket Bats
Cave paintings of Sonbhadra and Chitrakoot
Fabrics like chiffon, muslin and organza
Metalwork of Badohi and decorated terracotta horse

Solution: a)
Justification: Option A: These are some of the districts where Kasmir
willow bats are made. The willow used in making these bats was brought in
by the British. These willows are lighter and more powerful than ordinary
wood, making them more effective as a hitting bat.

INSIGHTS PRELIMS TEST SERIES 2017

www.insightsonindia.com

TEST 1 Solutions

www.insightsias.com

Option B: The cave paintings of Sonbhadra and Chitrakoot depict scenes of


hunting, war, festivals, dances, romantic life and animals and belong to UP.
Option C: One of the important crafts of Uttar Pradesh is Chikankari, which
entails delicate and traditional hand embroidery. This form of handicrafts
is mainly practiced in Lucknow. It is done on fabrics like chiffon, muslin,
organza, organdie and silk.
Option D: Gorakhpur has villages where clay figures of animals are done
and It is famous for its ornately decorated terracotta horse. The potter
creates the basic form by throwing separate pieces on the wheel and then
joining them.
Moradabad in U P is famous for art metalwork and known for its coloured
enamelling and intricate engravings.
Q Source: Improvisation: Page 75: Geography NCERT 7th: Our Environment




64.
Recently, India received the official citation from WHO and
UNICEF for being YAWS-free. Consider the following with reference
to this.
1. The Disease affects mainly the old age people due to lack of proper
immune response from the body.
2. India is the first country to be officially acknowledged as being
Yaws-free.
Which of the above is/are correct?
a)
b)
c)
d)

1 only
2 only
Both 1 and 2
None

Solution: b)

INSIGHTS PRELIMS TEST SERIES 2017

www.insightsonindia.com

TEST 1 Solutions

www.insightsias.com

Justification: Statement 1: It affects mainly children before the age of 15


years. 75-80% cases are related to children.
The disease occurs mainly in poor communities in warm, humid, tropical
areas where lack of proper water and sanitation facilities further help the
disease spread.
Statement 2: India has achieved this important milestone of being Yawsfree much before the WHO global target year of 2020.
Learning: Yaws is a chronic infection that affects mainly the skin, bone and
cartilage.
It is caused by the bacterium - Treponema pallidumm and transmitted by
skin contact.
Eradication of Yaws in India has been possible because of education and
early treatment of vulnerable population.
You can read more about it here
http://www.who.int/mediacentre/factsheets/fs316/en/
Q Source: Current Affairs: WHO Website:
http://www.who.int/neglected_diseases/news/India_global_eradication_y
aws_celebration/en/



65.
What is the Green corridor project that is recently being
pushed by state governments?
a) A project for responding to climate change by a combination of
adaptation and mitigation measures including carbon
sequestration.
b) A dedicated State Investment and Manufacturing Zone (SIMZ)
for establishment of Green Energy companies and services
c) A project for evacuation of renewable energy from generation
points to the load centres by creating intrastate and inter-state
transmission infrastructure.

INSIGHTS PRELIMS TEST SERIES 2017

www.insightsonindia.com

TEST 1 Solutions

www.insightsias.com

d) A project for protecting, restoring and enhancing Indias


diminishing forest cover by establishing inter-state buffer
protected areas

Solution: c)
Justification: Green Energy Corridor Project aims at synchronising
electricity produced from renewable sources, such as solar and wind, with
conventional power stations in the grid.
The government has taken lessons from the massive power grid
failure that hit the North, East and North-East regions of the country
in 2012.
Germany, who has expertise in making smart grids that integrate
renewable energy into national grid will be assisting India in this
project.
The intra-state transmission component of the project is being
implemented by the respective states and the Power Grid
Corporation of India (PGCIL) is implementing inter-state component.
Learning: PGCIL is setting up the first corridor connecting states rich in
renewable energy.
A second corridor would connect the solar parks in Andhra Pradesh,
Madhya Pradesh, Karnataka, Rajasthan and Gujarat.
The problem of voltage fluctuation in integrating conventional grids
with renewable energy grids would be taken care of by the German
technology and support.
Q Source: http://www.thehindu.com/news/national/tamil-nadu/indiasfirst-green-corridor-inaugurated/article8895647.ece


INSIGHTS PRELIMS TEST SERIES 2017

www.insightsonindia.com

TEST 1 Solutions

www.insightsias.com

66.
Which of the following are Permanent winds on the planet
earth?
a) Easterlies ad Westerlies
b) Monsoon winds
c) Continental winds
d) Orographic winds

Solution: a)
Learning: Winds can be broadly divided into three types.
Permanent winds The trade winds, westerlies and easterlies are the
permanent winds.
These blow constantly throughout the year in a particular direction.
Seasonal winds These winds change their direction in different
seasons. For example monsoons in India.
Local winds These blow only during a particular period of the day
or year in a small area. For example, land and sea breeze, and loo.


Q Source: Page 25: Geography NCERT 7th: Our Environment

INSIGHTS PRELIMS TEST SERIES 2017

www.insightsonindia.com

TEST 1 Solutions

www.insightsias.com







67.
Which of the following statements about the Troposphere
is/are NOT correct?
a) Almost all the weather phenomena like rainfall and hailstorm
occur in this layer.
b) It is the most ideal atmospheric layer for flying aeroplanes.
c) It helps in radio transmission as radio waves transmitted from
the earth are reflected back to the earth by this layer.
d) Both (b) and (c)

Solution: d)
Justification: Option A: Clouds and most other gases occur in this layer.
Movement of air mass (convection currents) within the layer gives rise to
weather phenomena.
Option B: It is stratosphere as this layer is almost free from clouds and
associated weather phenomenon, making conditions most ideal for flying
aeroplanes.
Option C: It is ionosphere which extends from 80-400 Km in height.
Learning: Exosphere: The upper most layer of the atmosphere is known as
exosphere. This layer has very thin air. Light gases like helium and
hydrogen float into the space from here.
Q Source: Page 23: Geography NCERT 7th: Our Environment

INSIGHTS PRELIMS TEST SERIES 2017

www.insightsonindia.com

TEST 1 Solutions

www.insightsias.com

68.
Many states in the Indian Union have instituted the post of
Parliamentary Secretary. A Parliament Secretary is
1. Generally a Civil Servant/bureaucrat holding an Office of Profit
2. Assigned charge of an entire government department only in the
absence of Cabinet Ministers
Which of the above is/are correct?
a)
b)
c)
d)

1 only
2 only
Both 1 and 2
None

Solution: d)
Justification & Learning: A Parliamentary Secretary often holds the rank
of Minister of State and has the same entitlements and is assigned to a
government department.
Manipur, HP, Mizoram, Assam, Rajasthan, Punjab, Goa are some of
the states where MLAs have been appointed Parliament Secretaries
by the Government.
Various petitions in the High Court have challenged the appointment
of Parliament Secretary, including in Delhi most recently.
In June 2015, Calcutta HC quashed appointment of 24 Parliamentary
Secretaries in West Bengal dubbing it unconstitutional.
Delhi High Court in 2016 quashed the appointment of Parliamentary
secretaries as they are deemed to be Ministers, and there is cap on the
number of ministers in the government as per the Constitution (The limit is
10% for Delhi, owing to its special status)
Q Source: http://timesofindia.indiatimes.com/india/Parliamentarysecretaries-de-facto-ministers-Law-ministry/articleshow/52755852.cms




INSIGHTS PRELIMS TEST SERIES 2017

www.insightsonindia.com

69.
at

TEST 1 Solutions

www.insightsias.com

The campaign 'Swachh Yug' launched by the government aims


a) Making Gram Panchayats located along Ganga open defecationfree
b) Rehabilitating manual scavengers in rural areas
c) Uplifting the moral standards in school and college education
d) Increasing the participation of youth and children in Swacch
Bharat Mission all over India

Solution: a)
Learning: It is a collaborative effort of three Union Ministries, to bring
about behavioural change among people staying in villages along the river
Ganga.
It covers five states - UP, Uttarakhand, Bihar, Jharkhand and West
Bengal from where the river flows.
A nodal officer has been identified for each district to work on
making area under their jurisdiction open defecation free (ODF) in a
"mission mode" and clean through proper solid and liquid waste
management.
In addition to monetary incentive offered under Swachh Bharat
Mission, extensive interpersonal behaviour change communication
training will be given to local trainers through network of virtual
classrooms.
Ministries involved in campaign
Ministry of Drinking Water and Sanitation mission mode strategy to
focus on cleanliness of village through proper solid and liquid waste
management.
Ministry of Youth Affairs and Sports will enlist support of youth
agencies like Bharat Scouts and Guides, Nehru Yuva Kendras and
National Service Scheme.
Ministry of Water Resources, River Development and Ganga
Rejuvenation

INSIGHTS PRELIMS TEST SERIES 2017

www.insightsonindia.com

TEST 1 Solutions

www.insightsias.com

Q Source: http://pib.nic.in/newsite/PrintRelease.aspx?relid=145999



70.
The Jatakas often shown on the railings of stupas and in
paintings in places such as Ajanta are actually?
a) Stories that tell about the previous lives of the Buddha
b) Essential teachings of the Buddha and his disciples
c) Cycles of birth and death as deciphered by the monks
d) Sangha rules that helped them travel and live together to
propagate dhamma

Solution: a)
Justification: Sangha rules are actually written in Vinaya pitaka, so D is
incorrect.
Buddhas teachings are given in Sutta Pitaka. So, B is incorrect.
Abhidhammapitaka deals with the philosophy and doctrine of
Buddhism appearing in the suttas.
Learning: Jataka stories are preserved in all branches of Buddhism. Some
Jataka tales are scattered in various sections of the Pali canon of Buddhist
writings,
The future Buddha may appear in them as a king, an outcast, a god,
an elephant - but, in whatever form, he exhibits some virtue that the
tale thereby inculcates.
Many Jatakas have parallels in the Mahabharata, the Panchatantra
(animal fables), the Puranas, and elsewhere in non-Buddhist Indian
literature.
Q Source: Page 129: NCERT 6th History: Our Pasts-I



INSIGHTS PRELIMS TEST SERIES 2017

www.insightsonindia.com

TEST 1 Solutions

www.insightsias.com


71.
Consider the following about Glaciers.
1. Glacial retreat can result in variety of landforms like lakes.
2. Materials deposited by a glacier as it retreats results in glacial
moraines.
Which of the above is/are correct?
a)
b)
c)
d)

1 only
2 only
Both 1 and 2
None

Solution: c)
Justification: Statement 1: The material carried by the glacier such as
rocks big and small, sand and silt gets deposited. These deposits form
glacial moraines.
Statement 2: Glaciers movements carve out deep hollows. When they
retreat and as the ice melts these hollows get filled up with water and
become beautiful lakes in the mountains.
You can read all about Glaciers and their features here
http://nationalgeographic.org/encyclopedia/glacier/
Q Source: Improvisation: Page 16: Geography NCERT 7th: Our Environment




72.
Krishi Kalyan Cess, to finance improvement of agriculture
and farmers welfare, is levied in India on
a) Services sector
b) Manufacturing Sector
c) Fertilizers and natural gas sector

INSIGHTS PRELIMS TEST SERIES 2017

www.insightsonindia.com

TEST 1 Solutions

www.insightsias.com

d) Groundwater irrigation equipments

Solution: a)
Learning: Imposition of Krishi Kalyan Cess of 0.5% was announced in the
Budget 2016-17 to come to force on 1st June 2016.
KKC is a cess, applicable on all services. It is to be solely used towards
financing activities for the improvement of agriculture and farmer
welfare.
Hence new effective Service Tax hence increased to 15% from 14.5%.
Budget target for Krishi Kalyan Cess is Rs. 5000 crore.
The KKC is based on the idea that levying a cess on a thriving sector
service sector (58% of GDP) of the economy can help fund a lifeline
to a sector that is in distress (Agriculture).
Q Source: http://indianexpress.com/article/business/budget/budget2016-govt-announces-0-5-pc-krishi-kalyan-cess-to-fund-farmprogrammes/



73.
A loan has been approved by an international agency for
constructing a bridge across Ganga in Bihar. How will the Ganga
Bridge be helpful?
1. It will provide vital transport link between Southern Uttar
Pradesh and Odisha.
2. It will improve Bihars connectivity with Nepal.
Which of the above is/are correct?
a)
b)
c)
d)

1 only
2 only
Both 1 and 2
None

INSIGHTS PRELIMS TEST SERIES 2017

www.insightsonindia.com

TEST 1 Solutions

www.insightsias.com

Solution: b)
Justification: Statement 1 and 2: The bridge would provide vital transport
links between the northern and southern parts of the state and with
neighbouring country Nepal.
The Ganga railroad bridge connects Digha Ghat in Patna and Pahleja
Ghat in Sonpur, Saran district in Bihar.
Regular scheduled passenger rail service was inaugurated on this
route on 3 February 2016.
This is second railway bridge in Bihar after Rajendra Setu that
connects North Bihar to South Bihar
Learning: Asian Development Bank (ADB) has recently approved $500
million (approximately Rs 3350 crore) loan for the bridge.
Once built, the 9.8 km road bridge in Bihar will be countrys longest river
bridge.
The project will run for about 4 years and is expected to complete by the
end of December 2020.
Q Source: http://www.hindustantimes.com/patna/foundation-stone-forbihar-bridge-laid-again-for-lalu-s-sake/storynEVZIGu0LWETQjQo20yaBP.html




74.
Consider the following about the Asian Development Bank
(ADB).
1. It was founded before World Bank (WB).
2. ADB assists its members by providing loans and technical
assistance to promote socio-economic development.
3. All its members are from the Asia-Pacific region.

INSIGHTS PRELIMS TEST SERIES 2017

www.insightsonindia.com

TEST 1 Solutions

www.insightsias.com

Select the correct answer using the codes below.


a)
b)
c)
d)

1 only
2 and 3 only
2 only
1 and 3 only

Solution: c)
Justification: Statement 1: The Asian Development Bank was conceived in
the early 1960s as a financial institution that would be Asian in character
and foster economic growth and cooperation in one of the poorest regions
in the world.
Statement 2: ADB assists its members (for e.g. India in infrastructure
projects), and partners, by providing loans, technical assistance, grants, and
equity investments to promote social and economic development.
Statement 3: ADB is composed of 67 members, 48 of which are from the
Asia and Pacific region. So, 3 is wrong.
India became a member of the Asian Development Bank (ADB) as a
founding member in 1966.
Japan and the US represent the largest shareholders.
Q Source: Ganga bridge loan by ADB



75.
Consider the following about the Nano Mission of the
Government of India.
1. The Department of Science and Technology is the nodal agency for
implementing the Nano Mission.
2. It can publish guidelines on Safe Handling of Nano-materials.
3. It funds all the private research agencies on development of nanomaterials.

INSIGHTS PRELIMS TEST SERIES 2017

www.insightsonindia.com

TEST 1 Solutions

www.insightsias.com

Select the correct answer using the codes below.


a)
b)
c)
d)

1 and 2 only
2 and 3 only
3 only
1 and 3 only

Solution: a)
Justification: Statement 1: The right answer can be reached by elimination
easily.
The Government of India, in 2007, had approved the launch of a Mission on
Nano Science and Technology (Nano Mission) with DST as the nodal body.
Statement 2: The Nano Mission has recently come out with the draft
Guidelines and Best Practices for Safe Handling of Nanomaterials in
Research Laboratories and Industries.
Statement 3: The Nano Mission will strive for development of products and
processes for national development, especially in areas of national
relevance like safe drinking water, materials development, sensors
development, drug delivery, etc.
It doesnt fund all private research in India. So, 3 is wrong.
We will be covering questions on the applications of nanotechnology in the
coming tests.
Q Source: http://dst.gov.in/scientific-programmes/mission-nano-scienceand-technology-nano-mission



76.
Consider the following statements about the Geography of
continents.
1. Asia is separated from Europe by the Alps Mountains on the west.
2. The Arctic Circle passes through Europe.

INSIGHTS PRELIMS TEST SERIES 2017

www.insightsonindia.com

TEST 1 Solutions

www.insightsias.com

3. North America is linked to South America by a narrow strait of


land.
Select the correct answer using the codes below.
a)
b)
c)
d)

1 and 2 only
2 and 3 only
1 and 3 only
1, 2 and 3

Solution: b)
Justification: Statement 1: The diagram below shows it. It is Ural
mountains.


Statement 2:

INSIGHTS PRELIMS TEST SERIES 2017

www.insightsonindia.com

TEST 1 Solutions

www.insightsias.com


Statement 3: It is the Isthmus of Panama that connects North with South
America.
Q Source: Page 32-33: NCERT 6th Geography: The Earth Our Habitat



77.
a)
b)
c)
d)

Dhamma Mahamattas were appointed by Ashoka to


Design a new religious code for the public
Administer all donations to religious organizations and trusts
in the empire
Teach people about Ashokas Dhamma
Represent the empire in religious debates with other empires
and eminent scholars

Solution: c)

INSIGHTS PRELIMS TEST SERIES 2017

www.insightsonindia.com

TEST 1 Solutions

www.insightsias.com

Justification: Ashoka instituted a body of officials known as the dhammamahamattas, who served the dual function of propagating the dhamma and
keeping the emperor in touch with public opinion
They were directly responsible for the practical working of Dhamma.
These officers directly reported to the king and they could have been
a 'privileged group'.
It may be assumed that gradually their power to interfere in the lives
of the people increased.
These officers were busy among members of various sects (edicts
mention that they worked among servants and nobles, Brahmans and
wealthy household-ers, the poor and the aged and others).
Learning: Ashokas dhamma did not involve worship of a god, or
performance of a sacrifice.
He felt that just as a father tries to teach his children, he had a duty to
instruct his subjects. He was also inspired by the teachings of the
Buddha.
Ashoka also sent messengers to spread ideas about dhamma to other
lands, such as Syria, Egypt, Greece and Sri Lanka
Q Source: Improvisation: Page 80: NCERT 6th History: Our Pasts-I




78.
a)
b)
c)
d)

Carbfix project is concerned with


Unearthing gas hydrates trapped at the Ocean floor
Locking away atmospheric CO2 by reacting it with basaltic
rocks
Reducing the methane emissions of rice fields and wetlands
Using geo-engineering to reduce the albedo of Polar Ice

INSIGHTS PRELIMS TEST SERIES 2017

www.insightsonindia.com

TEST 1 Solutions

www.insightsias.com

Solution: b)
Justification: It is a project in Iceland that aims to lock away CO2 by
reacting it with basaltic rocks.
Carbonated water is injected into the rocks so that it reacts with Calcium,
Magnesium or Silicate material present in Basaltic rocks. This is called
enhanced weathering.
Thus, the CO2 is captured permanently without releasing any harmful byproducts.
Learning: Recent reports show that the project was able to solidify 95% of
the injected 250 tonnes of CO2 into calcite in 2 years, using 25 tonnes of
water per tonne of CO2.
This is a significant achievement and offers hope for future.
Cost of the process is very high. Also, since the reactions are
exothermic; it is reversible if the rocks are heated.
Further, the pumping activity generates seismic activity.
Q Source: http://www.thehindu.com/sci-tech/science/scientists-turnco2-into-rock-to-combat-climate-change/article8715026.ece



79.
In the context of Ancient history of India, Gramabhojakas and
Grihapattis respectively were
a) Largest landlords and Head Judge of the village
b) Slaves and bonded labourers
c) Census official and Policeman of the village
d) Village headman and independent farmers

Solution: d)
Justification & Learning: In the northern part of the country, the village
headman was known as the grama bhojaka.

INSIGHTS PRELIMS TEST SERIES 2017

www.insightsonindia.com

TEST 1 Solutions

www.insightsias.com

Usually, men from the same family held the position for generations
(post was hereditary).
The grama bhojaka was often the largest landowner. Generally, he
had slaves and hired workers to cultivate the land.
Besides, as he was powerful, the king often used him to collect taxes
from the village. He also functioned as a judge, and sometimes as a
policeman.
Apart from the gramabhojaka, there were other independent
farmers, known as grihapatis, most of whom were smaller
landowners.
And then there were men and women such as the dasa karmakara,
who did not own land, and had to earn a living working on the fields
owned by others.
Q Source: Page 89: NCERT 6th History: Our Pasts-I




80.
a)
b)
c)
d)

The Hubble sequence is


The order of evolution of life on earth after the Big Bang
A system used for classifying galaxies
A code used by the International Space Station to communicate
back to earth
Made of all stars within the Goldilocks zone

Solution: b)
Learning: Hubbles scheme divides regular galaxies into 3 broad classes
ellipticals, lenticulars and spirals based on their visual appearance.
Although this scheme, also known as the Hubble tuning fork diagram, is
now considered somewhat too simple, the basic ideas still hold.

INSIGHTS PRELIMS TEST SERIES 2017

www.insightsonindia.com

TEST 1 Solutions

www.insightsias.com

For a time the Hubble tuning fork was thought to be an evolutionary


sequence - that galaxies might evolve from one type to another progressing
from left to right across the tuning-fork diagram.


Q Source: Current Affairs: NASA Website: https://www.nasa.gov/imagefeature/goddard/2016/nasas-hubble-spots-a-lopsided-lynx


81.
Consider the following statements.
1. Warm ocean currents generally originate near sub-tropical
regions and move towards equator.
2. The cold currents carry water from polar or higher latitudes to
lower latitudes.
3. The areas where the warm and cold currents meet are devoid of
nutrient cycling and thus generally poor in fisheries.
Select the correct answer using the codes below.
a)
b)
c)
d)

1 and 2 only
2 only
1 and 3 only
1, 2 and 3

Solution: b)

INSIGHTS PRELIMS TEST SERIES 2017

www.insightsonindia.com

TEST 1 Solutions

www.insightsias.com

Justification: Statement 1 and 2: Generally, the warm ocean currents


originate near the equator and move towards the poles. So, 1 is wrong.
The cold currents carry water from polar or higher latitudes to
tropical or lower latitudes. So, 2 is correct.
The Labrador Ocean current is cold current while the Gulf Stream is a
warm current.
The ocean current influence the temperature conditions of the area.
Warm currents bring about warm temperature over land surface, and
the opposite is also true.
Statement 3: They are the best fishing grounds due to recycling of
nutrients. Seas around Japan and the eastern coast of North America are
such examples. The areas where a warm and cold current meet also
experience foggy weather making it difficult for navigation.
Q Source: Page 36-37: Geography NCERT 7th: Our Environment



82.
The SDR is an international reserve asset, created by the IMF in
1969 to supplement its member countries official reserves. Consider
the following about Special Drawing Rights (SDR).
1. SDRs cannot be exchanged for freely usable currencies.
2. Value of a SDR is fixed on an annual basis and revised on the
discretion of IMF.
Which of the above is/are correct?
a)
b)
c)
d)

1 only
2 only
Both 1 and 2
None

Solution: d)

INSIGHTS PRELIMS TEST SERIES 2017

www.insightsonindia.com

TEST 1 Solutions

www.insightsias.com

Justification: Statement 1: It was created as a supplementary international


reserve asset.
SDRs can be exchanged for freely usable currencies. So, 1 is wrong.
The value of the SDR is currently based on a basket of four major
currencies: the U.S. dollar, euro, the Japanese yen, and pound sterling. The
basket will be expanded to include the Chinese renminbi (RMB) as the fifth
currency, effective October 1, 2016.
Statement 2: Its value is determined daily based on the currency values of
its constituent currencies.
The SDR is neither a currency, nor a claim on the IMF. Rather, it is a
potential claim on the freely usable currencies of IMF members.
Q Source: Often in news as it new SDR will be effective from October 2016



83.
a)
b)
c)
d)

The Painted Grey Ware culture (PGW) is an


Pre-Vedic age culture
Iron Age culture
Stone Age culture
Soanian culture

Solution: b)
Learning: It is an Iron Age culture of the Gangetic plain and the GhaggarHakra valley, lasting from roughly 1200 BCE to 600 BCE.
PGW are extremely fine to touch, with a nice, smooth surface.
Perhaps these were used on special occasions, for important people,
and to serve special food.
It is contemporary to, and a successor of the Black and red ware
culture.

INSIGHTS PRELIMS TEST SERIES 2017

www.insightsonindia.com

TEST 1 Solutions

www.insightsias.com

It is succeeded by Northern Black Polished Ware from c. 700-500


BCE, associated with the rise of the great mahajanapada states and of
the Magadha Empire.


Q Source: Page 56: NCERT 6th History: Our Pasts-I





84.
The Financial Stability Board (FSB) is an international body
that monitors and makes recommendations about the global financial
system. Consider the following about it.
1. It was established by G-20 as the successor to the Financial
Stability Forum (FSF).
2. It has been made a member organ of the International Monetary
and Financial Committee (IMFC).

INSIGHTS PRELIMS TEST SERIES 2017

www.insightsonindia.com

TEST 1 Solutions

www.insightsias.com

Which of the above is/are correct?


a)
b)
c)
d)

1 only
2 only
Both 1 and 2
None

Solution: a)
Justification: Statement 1: To strengthen the surveillance of financial
markets, the G20 leaders decided in April 2009 to expand the membership
of the former Financial Stability Forum (FSF) and renamed it the Financial
Stability Board (FSB). The new membership includes all G20 countries,
Hong Kong SAR, Singapore, Spain, and the European Commission.
Statement 2: FSB will report any possible threats to the stability of the
global financial system to the G20 finance ministers, the IMF and central
bank governors.
The FSF was made an observer of the IMFC in 1999. FSB is not a member
organ of the IMFC.
You can read more about it here http://www.fsb.org/about/
Q Source: Often in news:
http://www.imf.org/external/np/exr/facts/groups.htm#IC



85.
South west monsoon season is marked by the onset and
advance of monsoon. In this season the Monsoon winds blow from
a) Arabian Sea and Bay of Bengal towards the land
b) Only from Central Pacific towards Bay of Bengal
c) Land to the Bay of Bengal and Southern India
d) Only from Arabian Sea towards the land

Solution: a)

INSIGHTS PRELIMS TEST SERIES 2017

www.insightsonindia.com

TEST 1 Solutions

www.insightsias.com

Justification: These Monsoon winds lift moisture from both these oceans
and carry moisture with them.
When these winds strike the mountain barriers (say in Himalayas or Northeastern Mountains), rainfall occurs.
We will be covering other patterns of wind and ocean current circulation
like El-Nino (that affect Monsoon) in later tests.
Q Source: Page 57: NCERT 6th Geography: The Earth Our Habitat



86.
If the earth starts rotating east to west, which of the following
shall be correct?
1. Places West of Greenwich will be behind the Greenwich Time.
2. Places located on the same longitude northwards will be ahead in
time than those located southwards.
Which of the above is/are correct?
a)
b)
c)
d)

1 only
2 only
Both 1 and 2
None

Solution: d)
Justification: Statement 1: It is the case presently when earth rotates from
west to east. If the opposite happens, places west of Greenwich will be
ahead in time and places east will lag behind in time.
Statement 2: Time varies across a longitude, not latitude.
For e.g. when the Prime Meridian of Greenwich has the sun at the highest
point in the sky, all the places along this meridian will have mid-day or
noon, whether the place is southwards or northwards.

INSIGHTS PRELIMS TEST SERIES 2017

www.insightsonindia.com

TEST 1 Solutions

www.insightsias.com

This is independent of the direction of rotation of the earth. So, 2 is


incorrect.
Q Source: Page 15: NCERT 6th Geography: The Earth Our Habitat




87.
The Indian Railways plans to introduce Maglev trains that can
run at a top speed of 500 km an hour, in a bid to lure passengers from
airlines. Maglev technology harnesses the power of
a) Magnetic fields to propel train at high speed
b) Earths gravitational force
c) Wind in a small tunnel to transfer momentum to the train
d) Solar PV technology to increase the burn rate of fuel in the train
engine

Solution: a)
Learning: Maglev (derived from magnetic levitation) is a transport method
that uses magnetic levitation to move vehicles without touching the
ground.
With maglev, a vehicle travels along a guideway using magnets to create
both lift and propulsion, thereby reducing friction by a great extent and
allowing very high speeds.
Maglev trains are in operation in China, Japan, Germany and South Korea.
As the trains are propelled by magnetic forces, friction is eliminated,
making transportation free of noise and vibration.
Q Source: http://www.thehindu.com/news/national/fly-on-rails-globaltenders-floated-to-induct-maglev-trains/article8949604.ece

INSIGHTS PRELIMS TEST SERIES 2017

www.insightsonindia.com

TEST 1 Solutions

www.insightsias.com


88.
a)
b)
c)
d)

ScatSat-1 satellite recently launched by ISRO will help in


Mineral exploration
Forecasting groundwater resources in India
Increasing the bandwidth of telecommunication services in
India
Disaster management

Solution: d)
Learning: Currently India is dependent on NASAs ISS-RapidScat for
prediction of cyclone forecasting and weather prediction.
With launch of this satellite India will acquire the capability for more
accurate prediction and tracking of cyclones.
The indigenously developed weather forecasting satellite will be
equipped with on board instruments to monitor sea surface winds
and help predict the genesis of cyclones
The primary payload of ScatSat-1 is a scatterometer to keep a watch
on the speed and direction of ocean winds that indicate the formation
and strengthening of cyclones.
The satellite will have the capacity to monitor temperature and
humidity.
Q Source: http://www.thehindu.com/todays-paper/tp-national/scatsat-1to-help-in-predicting-cyclones/article8945917.ece




89.
Consider the following statements about the geography of
India.

INSIGHTS PRELIMS TEST SERIES 2017

www.insightsonindia.com

TEST 1 Solutions

www.insightsias.com

Assertion (A): East-West extent of India is greater than its North-South


Extent.
Reason (R): India is located entirely in the Northern Hemisphere.
In the context of the above, which of these is correct?
a)
b)
c)
d)

A is correct, and R is an appropriate explanation of A.


A is correct, but R is not an appropriate explanation of A.
A is incorrect, but R is correct.
Both A and R are incorrect.

Solution: c)
Justification: From south to north, India extends between 84'N and
376'N latitudes. From west to east, India extends between 687'E and
9725'E longitudes.
So, the north-south extent from Kashmir to Kanyakumari is about 3,200
km. And the east-west extent from Arunachal Pradesh to Kuchchh is about
2,900 km.
It has nothing to do with which hemisphere India lies in.
Q Source: Page 47: NCERT 6th Geography: The Earth Our Habitat



90.
Which of the following generally does NOT come within the
functional domain of the Government in India?
a) Maintaining public roads
b) Establishing diplomatic relations with other states
c) Introduction of new railways lines between remote locations
d) Producing goods of daily consumption

Solution: d)
Justification: You need to choose most appropriate option out of them.

INSIGHTS PRELIMS TEST SERIES 2017

www.insightsonindia.com

TEST 1 Solutions

www.insightsias.com

Option A: NHAI, Ministry of Road Transport etc take care of public roads,
national highways in India.
Option B: Ministry of External affairs maintains diplomatic relations with
other states.
Option C: It is done by Indian Railways on proposals from the Ministry of
Railways.
Option D: Some PSUs do produce goods/commodities of daily
consumption like petrol and Diesel, but this is not the general case.
Production of most other articles is left to the private sector, like salt,
clothes etc.
Q Source: Page 28: NCERT 6th: Social and Economic Life I


91.
Only one side of the moon is visible to us on the earth mainly
due to
1. Spin speed of the Moon on its axis which matches with that of
earth
2. Geoid shape and axial tilt of the earth
3. Atmospheric refraction on earth and absence of atmosphere in
space
Select the correct answer using the codes below.
a)
b)
c)
d)

1 only
1 and 3 only
1 and 2 only
2 and 3 only

Solution: a)
Justification: Statement 1: The moon moves around the earth in about 27
days. It takes exactly the same time to complete one spin. As a result, only
one side of the moon is visible to us on the earth. You can watch this

INSIGHTS PRELIMS TEST SERIES 2017

www.insightsonindia.com

TEST 1 Solutions

www.insightsias.com

animation for better understanding


https://www.youtube.com/watch?v=OZIB_leg75Q
Statement 2: Even if the earth was not tilted, you would be able to see only
one side of the moon. The geoid shape of earth has nothing to do with our
view of the Moon. Even if the earth were to become slightly flatter or
rounder, the sight of the moon wont change much.
Statement 3: If at all this was applicable, it should have been applicable to
all cosmic bodies, which is not true. Atmospheric refraction helps planets,
stars twinkle and doesnt decide which side of them is visible or invisible
on earth.
Q Source: Page 5: NCERT 6th Geography: The Earth Our Habitat



92.
The Food Safety and Standards Authority of India (FSSAI) has
been regulating the use of harmful trans fats in packaged food items.
Consider the following about trans fats.
1. They are different from ordinary fats in that they are naturally not
found or formed in animal or plant products.
2. They are used by food manufacturers to improve the texture and
shelf life of food.
3. Eating them raises the level of low-density lipoprotein (LDL)
cholesterol in the blood.
Select the correct answer using the codes below.
a)
b)
c)
d)

2 and 3 only
1 and 2 only
1 and 3 only
1, 2 and 3

Solution: a)

INSIGHTS PRELIMS TEST SERIES 2017

www.insightsonindia.com

TEST 1 Solutions

www.insightsias.com

Justification: Statement 1: There are two sources of trans fat, also known
as trans fatty acids:
Trans fat formed naturally, produced in the gut of some grazing
animals as also found in animal products like meat, milk, and milk
products.
Trans fat formed during food processing this type of trans fat is
created when hydrogen is added to vegetable oil (a process called
hydrogenation) to make it more solid.
Statement 2: Partially hydrogenated oils are used by food manufacturers
to improve the texture, shelf life, and flavor stability of foods.
Statement 3: Trans fat can be found in many of the same foods as saturated
fat. These can include Coffee creamer, Crackers, cookies, cakes, frozen pies
etc.
An elevated LDL (bad) blood cholesterol level can increase your risk of
developing cardiovascular disease. HDL is good blood cholesterol.
Q Source: Improvisation: http://www.downtoearth.org.in/news/foodsafety-agency-extends-implementation-of-trans-fats-standards-by-sixmonths-55274


93.
As per the 73rd Constitutional Amendment Act 1992, "Gram
Sabha" means a body consisting of persons
a) Registered in the electoral rolls relating to a village comprised
within the area of Panchayat at the village level
b) Residing in the village or panchayat or contiguous areas by way
of naturalization of migration
c) Specified by the Governor by a public notification on the
recommendation of the State Election Commission
d) Chosen so by the Panchayat to address the socio-economic
issues of the village

Solution: a)

INSIGHTS PRELIMS TEST SERIES 2017

www.insightsonindia.com

TEST 1 Solutions

www.insightsias.com

Learning: As per the 73rd Amendment Act 1992, a Gram Sabha may
exercise such powers and perform such functions at the village level as the
Legislature of a State may, by law, provide.
There shall be constituted in every State, Panchayats at the village,
intermediate and district levels in accordance with the provisions of this
Part.
Subject to the provisions of this Part, the Legislature of a State may, by law,
make provisions with respect to the composition of Panchayats.
We will be covering more question later on the powers and role of the
Gram Sabha in a village.
Q Source: Page 43: NCERT 6th: Social and Economic Life I



94.
Arrange the following from East to West.
1. Mahadeo Hills
2. Rajmahal Hills
3. Aravalli range
Select the correct answer using the codes below.
a)
b)
c)
d)

213
312
123
231

Solution: a)
Justification: The map below is self-explanatory.

INSIGHTS PRELIMS TEST SERIES 2017

www.insightsonindia.com

TEST 1 Solutions

www.insightsias.com


Q Source: Map based questions: India



95.
Fiscal Monitor report was published in April 2016 by which
of these organizations?
a) World Bank
b) World Economic Forum
c) International Monetary Fund
d) Reserve Bank of India

Solution: c)
Learning: The Fiscal Monitor was launched in 2009 to survey and analyze
the latest public finance developments and other aspects related to it.
It is prepared twice a year by the IMFs Fiscal Affairs Department.
Its projections are based on the same database used for the World
Economic Outlook (WEO) and the Global Financial Stability Report
(GFSR).
It is important that the IMF monitors the fiscal situation of its
member nations as it acts a lender of last resort in case of BoP crisis.

INSIGHTS PRELIMS TEST SERIES 2017

www.insightsonindia.com

TEST 1 Solutions

www.insightsias.com

Q Source: Current Affairs: IMF Website






96.
Scientists have recently developed a new bio-ink that allows 3D
printing of complex living tissues that may be used for surgical
implants. The bio-ink contains which of the following?
1. Natural polymer extracted from seaweed
2. Stem cells that have potency and unspecialized states
Which of the above is/are correct?
a)
b)
c)
d)

1 only
2 only
Both 1 and 2
None

Solution: c)
Justification: Statement 1: The bio-ink contains two different polymer
components: a natural polymer extracted from seaweed, and a sacrificial
synthetic polymer used in the medical industry.
The synthetic polymer causes the bio-ink to change from liquid to solid
when the temperature is raised, and the seaweed polymer provides
structural support when the cell nutrients are introduced.
Statement 2: Stem cells differ from other kinds of cells in the body. All stem
cells-regardless of their source - have three general properties: they are
capable of dividing and renewing themselves for long periods; they are
unspecialized; and they can give rise to specialized cell types.
They are crucial for organ transplants as they can regenerate into different
types of human organs.

INSIGHTS PRELIMS TEST SERIES 2017

www.insightsonindia.com

TEST 1 Solutions

www.insightsias.com

Learning: You can learn about 3-D printing here


http://3dprinting.com/what-is-3d-printing/
Q Source:
http://indianexpress.com/article/technology/science/scientists-developnew-bio-ink-for-3d-printing-2874908/




97.
a)
b)
c)

d)

How is the continental crust different from oceanic crust?


Mineral constituents of the continental mass are Silica and
alumina, whereas that of oceanic crust is mainly iron
Continental crust is less dense than oceanic crust.
Continental crust has a highly diverse composition of elements
in both fused and natural form, whereas oceanic crust is made
from a single element only
All of the above

Solution: b)
Justification: Option A: The main mineral constituents of the continental
mass are silica and alumina. The oceanic crust mainly consists of silica and
magnesium. So, A is wrong.
It is core that is largely made of iron.
Option B: Both oceanic crust and continental crust are less dense than the
mantle, but oceanic crust is denser than continental crust. This is partly
why the continents are at a higher elevation than the ocean floor.
Because continental crust is less dense than oceanic crust it floats higher on
the mantle, just like a piece of Styrofoam floats higher on water than a piece
of wood does.
Option C: It is not true, as also mentioned in option A justification.

INSIGHTS PRELIMS TEST SERIES 2017

www.insightsonindia.com

TEST 1 Solutions

www.insightsias.com

Q Source: Page 8: Geography NCERT 7th: Our Environment





98.

Consider the following statements.

Assertion (A): Chief Minister of the State can disqualify a sitting MLA after
obtaining the advice of the State Election Commission.
Reason (R): Chief Minister holds the de facto authority in the State
Government.
In the context of the above, which of these is correct?
a)
b)
c)
d)

A is correct, and R is an appropriate explanation of A.


A is correct, but R is not an appropriate explanation of A.
A is incorrect, but R is correct.
A is correct, but R is incorrect.

Solution: c)
Justification: An MLA can be disqualified on many grounds.
On grounds of defection (to be covered later in tests), the Speaker
decides the disqualification.
If he holds any office of profit under the Government of India or a
state or an office declared by a law of the state, he can be disqualified.
If any competent court declares any member to be of unsound mind,
then the MLA is disqualified.
Under Article 192 of the Constitution, if any question arises as to
whether or not the member of a house of the legislature of a state has
become subject to any of the disqualification criteria, the question
shall be cited to the Governor of the State for decision who will act as
per the opinion of the Election Commission (EC).

INSIGHTS PRELIMS TEST SERIES 2017

www.insightsonindia.com

TEST 1 Solutions

www.insightsias.com

His decision shall be final and may not be reviewed by any court of
law. So, clearly A is wrong.
Q Source: Improvisation: Page 33: NCERT 7th: Social and Economic Life II



99.
A Bengali writer, her book titled Amar Jiban is the first known
autobiography written by an Indian woman. She is?
a) Ismat Chugtai
b) Begum Rukhaiya Hussain
c) Deepa Bandopadhyay
d) Rashsundari Devi

Solution: d)
Learning: At the age of 60, Rashsundari Devi wrote her autobiography in
Bangla in 1860. She and a few others like (Rukhaiya Hussain) were some
the most prominent figures in the literary history of women in colonial
India.
Option A: Ismat Chugtai (post-colonial period) was a liberal Muslim writer
and considered to be a path-breaking woman writer of the region.
Option B: Begum Rukhaiya Hussain wrote Sultanas Dream in 1905 to
practise her English skills when she was merely 25 years old. This story
imagined a woman called Sultana who reaches a place called Ladyland.
Ladyland is a place where women took similar roles as men.
Q Source: Page 60: NCERT 7th: Social and Economic Life II


100.
As per a report commissioned by the Department of Science
and Technology (DST), India will be heavily dependent on China in

INSIGHTS PRELIMS TEST SERIES 2017

www.insightsonindia.com

TEST 1 Solutions

www.insightsias.com

the coming years to source rare earth minerals for their use in
manufacturing sector. What is the reason behind it?
a) India does not have any reserve of rare-earth minerals.
b) Sensitive imports from Missile Technology Control Regime
(MTCR) nations are banned for a non-member like India.
c) India does not have the necessary technology to extract any
rare-earth minerals.
d) China is a leading global supplier for many rare earth mineral
resources that have been identified as critical for India.

Solution: d)
Justification: Option A and C: India is endowed with vast mineral
resources - it is among the top five nations with reserves of rare-earth
minerals - its potential is untapped.
India currently has a little over 2% share of global output of rare
earths. Their production has also increased post-1995. But, more
than 97% supply lies with China.
Option B: These items are not in MTCR lists. MTCR list covers ballistic
missile technology.
Option D: China currently accounts for an overwhelming 97% of global
production, and it has not been shy of using this dominance as a bargaining
chip against other countries.
Rare earths are vital future resources and India wont like any
country to have a monopoly. So, it is planning to negotiate contracts
for future supplies.
Learning: 12 minerals out of 49 that were evaluated as most critical for
Indias manufacturing sector by 2030.
These are beryllium, chromium, germanium, limestone, niobium,
graphite, rare earths, rhenium, strontium, tantalum and zirconium.
These elements are critical to many industries of the futurefrom
smart phones to hybrid cars to solar panels.

INSIGHTS PRELIMS TEST SERIES 2017

www.insightsonindia.com

TEST 1 Solutions

www.insightsias.com

There is a worldwide scramble to get hold of rare earths. Prices have


soared in recent times.
India is 100 per cent import-dependent for seven out of 12 identified
critical minerals and does not have any declared resources for them,
except light rare-earths (found along with monazite sands) and
beryllium.
You can read more here
http://www.insightsonindia.com/2016/08/05/insights-daily-currentaffairs-05-august-2016/
Q Source: As given above




INSIGHTS PRELIMS TEST SERIES 2017

You might also like